Location via proxy:   [ UP ]  
[Report a bug]   [Manage cookies]                

GERD

Download as docx, pdf, or txt
Download as docx, pdf, or txt
You are on page 1of 69

1. Nurse Berlinda is assigned to a 41-year-old client who has a diagnosis of chronic pancreatitis.

The nurse reviews the laboratory result, anticipating a laboratory report that indicates a serum
amylase level of:

A. 45 units/L
B. 100 units/L
C. 300 units/L
D. 500 units/L
2. A male client who is recovering from surgery has been advanced from a clear liquid diet to a full
liquid diet. The client is looking forward to the diet change because he has been “bored” with the
clear liquid diet. The nurse would offer which full liquid item to the client?

A. Tea
B. Gelatin
C. Custard
D. Popsicle
3. Nurse Juvy is caring for a client with cirrhosis of the liver. To minimize the effects of the
disorder, the nurse teaches the client about foods that are high in thiamine. The nurse determines
that the client has the best understanding of the dietary measures to follow if the client states an
intension to increase the intake of:

A. Pork
B. Milk
C. Chicken
D. Broccoli
4. Nurse Oliver checks for residual before administering a bolus tube feeding to a client with a
nasogastric tube and obtains a residual amount of 150 mL. What is appropriate action for the nurse
to take?

A. Hold the feeding


B. Reinstill the amount and continue with administering the feeding
C. Elevate the client’s head at least 45 degrees and administer the feeding
D. Discard the residual amount and proceed with administering the feeding
5. A nurse is inserting a nasogastric tube in an adult male client. During the procedure, the client
begins to cough and has difficulty breathing. Which of the following is the appropriate nursing
action?

A. Quickly insert the tube


B. Notify the physician immediately
C. Remove the tube and reinsert when the respiratory distress subsides
D. Pull back on the tube and wait until the respiratory distress subsides
6. Nurse Ryan is assessing for correct placement of a nosogartric tube. The nurse aspirates the
stomach contents and check the contents for pH. The nurse verifies correct tube placement if which
pH value is noted?
A. 3.5
B. 7.0
C. 7.35
D. 7.5
7. A nurse is preparing to remove a nasogartric tube from a female client. The nurse should instruct
the client to do which of the following just before the nurse removes the tube?

A. Exhale
B. Inhale and exhale quickly
C. Take and hold a deep breath
D. Perform a Valsalva maneuver
8. Nurse Joy is preparing to administer medication through a nasogastric tube that is connected to
suction. To administer the medication, the nurse would:

A. Position the client supine to assist in medication absorption


B. Aspirate the nasogastric tube after medication administration to maintain patency
C. Clamp the nasogastric tube for 30 minutes following administration of the medication
D. Change the suction setting to low intermittent suction for 30 minutes after medication
administration
9. A nurse is preparing to care for a female client with esophageal varices who has just has a
Sengstaken-Blakemore tube inserted. The nurse gathers supplies, knowing that which of the
following items must be kept at the bedside at all times?

A. An obturator
B. Kelly clamp
C. An irrigation set
D. A pair of scissors
10. Dr. Smith has determined that the client with hepatitis has contracted the infection form
contaminated food. The nurse understands that this client is most likely experiencing what type of
hepatitis?

A. Hepatitis A
B. Hepatitis B
C. Hepatitis C
D. Hepatitis D
11. A client is suspected of having hepatitis. Which diagnostic test result will assist in confirming
this diagnosis?

A. Elevated hemoglobin level


B. Elevated serum bilirubin level
C. Elevated blood urea nitrogen level
D. Decreased erythrocycle sedimentation rate
12. The nurse is reviewing the physician’s orders written for a male client admitted to the hospital
with acute pancreatitis. Which physician order should the nurse question if noted on the client’s
chart?
A. NPO status
B. Nasogastric tube inserted
C. Morphine sulfate for pain
D. An anticholinergic medication
13. A female client being seen in a physician’s office has just been scheduled for a barium swallow
the next day. The nurse writes down which instruction for the client to follow before the test?

A. Fast for 8 hours before the test


B. Eat a regular supper and breakfast
C. Continue to take all oral medications as scheduled
D. Monitor own bowel movement pattern for constipation
14. The nurse is performing an abdominal assessment and inspects the skin of the abdomen. The
nurse performs which assessment technique next?

A. Palpates the abdomen for size


B. Palpates the liver at the right rib margin
C. Listens to bowel sounds in all for quadrants
D. Percusses the right lower abdominal quadrant
15. Polyethylene glycol-electrlyte solution (GoLYTELY) is prescribed for the female client
scheduled for a colonoscopy. The client begins to experience diarrhea following administration of
the solution. What action by the nurse is appropriate?

A. Start an IV infusion
B. Administer an enema
C. Cancel the diagnostic test
D. Explain that diarrhea is expected
16. The nurse is caring for a male client with a diagnosis of chronic gastritis. The nurse monitors
the client knowing that this client is at risk for which vitamin deficiency?

A. Vitamin A
B. Vitamin B12
C. Vitamin C
D. Vitamin E
17. The nurse is reviewing the medication record of a female client with acute gastritis. Which
medication, if noted on the client’s record, would the nurse question?

A. Digoxin (Lanoxin)
B. Furosemide (Lasix)
C. Indomethacin (Indocin)
D. Propranolol hydrochloride (Inderal)
18. The nurse is assessing a male client 24 hours following a cholecystectomy. The nurse noted that
the T tube has drained 750 mL of green-brown drainage since the surgery. Which nursing
intervention is appropriate?

A. Clamp the T tube


B. Irrigate the T tube
C. Notify the physician
D. Document the findings
19. The nurse is monitoring a female client with a diagnosis of peptic ulcer. Which assessment
findings would most likely indicate perforation of the ulcer?

A. Bradycardia
B. Numbness in the legs
C. Nausea and vomiting
D. A rigid, board-like abdomen
20. A male client with a peptic ulcer is scheduled for a vagotomy and the client asks the nurse
about the purpose of this procedure. Which response by the nurse best describes the purpose of a
vagotomy?

A. Halts stress reactions


B. Heals the gastric mucosa
C. Reduces the stimulus to acid secretions
D. Decreases food absorption in the stomach
21. The nurse is caring for a female client following a Billroth II procedure. Which postoperative
order should the nurse question and verify?

A. Leg exercises
B. Early ambulation
C. Irrigating the nasogastric tube
D. Coughing and deep-breathing exercises
22. The nurse is providing discharge instructions to a male client following gastrectomy and
instructs the client to take which measure to assist in preventing dumping syndrome?

A. Ambulate following a meal


B. Eat high carbohydrate foods
C. Limit the fluid taken with meal
D. Sit in a high-Fowler’s position during meals
23. The nurse is monitoring a female client for the early signs and symptoms of dumping
syndrome. Which of the following indicate this occurrence?

A. Sweating and pallor


B. Bradycardia and indigestion
C. Double vision and chest pain
D. Abdominal cramping and pain
24. The nurse is preparing a discharge teaching plan for the male client who had umbilical hernia
repair. What should the nurse include in the plan?

A. Irrigating the drain


B. Avoiding coughing
C. Maintaining bed rest
D. Restricting pain medication
25. The nurse is instructing the male client who has an inguinal hernia repair how to reduce
postoperative swelling following the procedure. What should the nurse tell the client?

A. Limit oral fluid


B. Elevate the scrotum
C. Apply heat to the abdomen
D. Remain in a low-fiber diet
26. The nurse is caring for a hospitalized female client with a diagnosis of ulcerative colitis. Which
finding, if noted on assessment of the client, would the nurse report to the physician?

A. Hypotension
B. Bloody diarrhea
C. Rebound tenderness
D. A hemoglobin level of 12 mg/dL
27. The nurse is caring for a male client postoperatively following creation of a colostomy. Which
nursing diagnosis should the nurse include in the plan of care?

A. Sexual dysfunction
B. Body image, disturbed
C. Fear related to poor prognosis
D. Nutrition: more than body requirements, imbalanced
28. The nurse is reviewing the record of a female client with Crohn’s disease. Which stool
characteristics should the nurse expect to note documented in the client’s record?

A. Diarrhea
B. Chronic constipation
C. Constipation alternating with diarrhea
D. Stools constantly oozing form the rectum
29. The nurse is performing a colostomy irrigation on a male client. During the irrigation, the client
begins to complain of abdominal cramps. What is the appropriate nursing action?

A. Notify the physician


B. Stop the irrigation temporarily
C. Increase the height of the irrigation
D. Medicate for pain and resume the irrigation
30. The nurse is teaching a female client how to perform a colostomy irrigation. To enhance the
effectiveness of the irrigation and fecal returns, what measure should the nurse instruct the client to
do?

A. Increase fluid intake


B. Place heat on the abdomen
C. Perform the irrigation in the evening
D. Reduce the amount of irrigation solution
31. A patient with chronic alcohol abuse is admitted with liver failure. You closely monitor the
patient’s blood pressure because of which change that is associated with the liver failure?
A. Hypoalbuminemia
B. Increased capillary permeability
C. Abnormal peripheral vasodilation
D. Excess rennin release from the kidneys
32. You’re assessing the stoma of a patient with a healthy, well-healed colostomy. You expect the
stoma to appear:

A. Pale, pink and moist


B. Red and moist
C. Dark or purple colored
D. Dry and black
33. You’re caring for a patient with a sigmoid colostomy. The stool from this colostomy is:

A. Formed
B. Semisolid
C. Semiliquid
D. Watery
34. You’re advising a 21 y.o. with a colostomy who reports problems with flatus. What food
should you recommend?

A. Peas
B. Cabbage
C. Broccoli
D. Yogurt
35. You have to teach ostomy self care to a patient with a colostomy. You tell the patient to
measure and cut the wafer:

A. To the exact size of the stoma.


B. About 1/16” larger than the stoma.
C. About 1/8” larger than the stoma.
D. About 1/4″ larger than the stoma.
36. You’re performing an abdominal assessment on Brent who is 52 y.o. In which order do you
proceed?

A. Observation, percussion, palpation, auscultation


B. Observation, auscultation, percussion, palpation
C. Percussion, palpation, auscultation, observation
D. Palpation, percussion, observation, auscultation
37. You’re doing preoperative teaching with Gertrude who has ulcerative colitis who needs
surgery to create an ileoanal reservoir. Which information do you include?

A. A reservoir is created that exits through the abdominal wall.


B. A second surgery is required 12 months after the first surgery.
C. A permanent ileostomy is created.
D. The surgery occurs in two stages.
38. You’re caring for Carin who has just had ileostomy surgery. During the first 24 hours post-op,
how much drainage can you expect from the ileostomy?

A. 100 ml
B. 500 ml
C. 1500 ml
D. 5000 ml
39. You’re preparing a teaching plan for a 27 y.o. named Jeff who underwent surgery to close a
temporary ileostomy. Which nutritional guideline do you include in this plan?

A. There is no need to change eating habits.


B. Eat six small meals a day.
C. Eat the largest meal in the evening.
D. Restrict fluid intake.
40. Arthur has a family history of colon cancer and is scheduled to have a sigmoidoscopy. He is
crying as he tells you, “I know that I have colon cancer, too.” Which response is most therapeutic?

A. “I know just how you feel.”


B. “You seem upset.”
C. “Oh, don’t worry about it, everything will be just fine.”
D. “Why do you think you have cancer?”
41. You’re caring for Beth who underwent a Billroth II procedure (surgical removal of the pylorus
and duodenum) for treatment of a peptic ulcer. Which findings suggest that the patient is
developing dumping syndrome, a complication associated with this procedure?

A. Flushed, dry skin.


B. Headache and bradycardia.
C. Dizziness and sweating.
D. Dyspnea and chest pain.
42. You’re developing the plan of care for a patient experiencing dumping syndrome after a
Billroth II procedure. Which dietary instructions do you include?

A. Omit fluids with meals.


B. Increase carbohydrate intake.
C. Decrease protein intake.
D. Decrease fat intake.
43. You’re caring for Lewis, a 67 y.o. patient with liver cirrhosis who develops ascites and
requires paracentesis. Relief of which symptom indicated that the paracentesis was effective?

A. Pruritus
B. Dyspnea
C. Jaundice
D. Peripheral Neuropathy
44. You’re caring for Jane, a 57 y.o. patient with liver cirrhosis who develops ascites and requires
paracentesis. Before her paracentesis, you instruct her to:
A. Empty her bladder.
B. Lie supine in bed.
C. Remain NPO for 4 hours.
D. Clean her bowels with an enema.
45. After abdominal surgery, your patient has a severe coughing episode that causes wound
evisceration. In addition to calling the doctor, which intervention is most appropriate?

A. Irrigate the wound & organs with Betadine.


B. Cover the wound with a saline soaked sterile dressing.
C. Apply a dry sterile dressing & binder.
D. Push the organs back & cover with moist sterile dressings.
46. You’re caring for Betty with liver cirrhosis. Which of the following assessment findings leads
you to suspect hepatic encephalopathy in her?

A. Asterixis
B. Chvostek’s sign
C. Trousseau’s sign
D. Hepatojugular reflex
47. You are developing a careplan on Sally, a 67 y.o. patient with hepatic encephalopathy. Which
of the following do you include?

A. Administering a lactulose enema as ordered.


B. Encouraging a protein-rich diet.
C. Administering sedatives, as necessary.
D. Encouraging ambulation at least four times a day.
48. You have a patient with achalasia (incomplete muscle relaxtion of the GI tract, especially
sphincter muscles). Which medications do you anticipate to administer?

A. Isosorbide dinitrate (Isordil)


B. Digoxin (Lanoxin)
C. Captopril (Capoten)
D. Propanolol (Inderal)
49. The student nurse is preparing a teaching care plan to help improve nutrition in a patient with
achalasia. You include which of the following:

A. Swallow foods while leaning forward.


B. Omit fluids at mealtimes.
C. Eat meals sitting upright.
D. Avoid soft and semisoft foods.
50. Britney, a 20 y.o. student is admitted with acute pancreatitis. Which laboratory findings do you
expect to be abnormal for this patient?

A. Serum creatinine and BUN


B. Alanine aminotransferase (ALT) and aspartate aminotransferase (AST)
C. Serum amylase and lipase
D. Cardiac enzymes
51. A patient with Crohn’s disease is admitted after 4 days of diarrhea. Which of the following
urine specific gravity values do you expect to find in this patient?

A. 1.005
B. 1.011
C. 1.020
D. 1.030
52. Your goal is to minimize David’s risk of complications after a heriorrhaphy. You instruct the
patient to:

A. Avoid the use of pain medication.


B. Cough and deep breathe Q2H.
C. Splint the incision if he can’t avoid sneezing or coughing.
D. Apply heat to scrotal swelling.
53. Janice is waiting for discharge instructions after her herniorrhaphy. Which of the following
instructions do you include?

A. Eat a low-fiber diet.


B. Resume heavy lifting in 2 weeks.
C. Lose weight, if obese.
D. Resume sexual activity once discomfort is gone.
54. Develop a teaching care plan for Angie who is about to undergo a liver biopsy. Which of the
following points do you include?

A. “You’ll need to lie on your stomach during the test.”


B. “You’ll need to lie on your right side after the test.”
C. “During the biopsy you’ll be asked to exhale deeply and hold it.”
D. “The biopsy is performed under general anesthesia.”
55. Stephen is a 62 y.o. patient that has had a liver biopsy. Which of the following groups of signs
alert you to a possible pneumothorax?

A. Dyspnea and reduced or absent breath sounds over the right lung
B. Tachycardia, hypotension, and cool, clammy skin
C. Fever, rebound tenderness, and abdominal rigidity
D. Redness, warmth, and drainage at the biopsy site
56. Michael, a 42 y.o. man is admitted to the med-surg floor with a diagnosis of acute pancreatitis.
His BP is 136/76, pulse 96, Resps 22 and temp 101. His past history includes hyperlipidemia and
alcohol abuse. The doctor prescribes an NG tube. Before inserting the tube, you explain the purpose
to patient. Which of the following is a most accurate explanation?

A. “It empties the stomach of fluids and gas.”


B. “It prevents spasms at the sphincter of Oddi.”
C. “It prevents air from forming in the small intestine and large intestine.”
D. “It removes bile from the gallbladder.”
57. Jason, a 22 y.o. accident victim, requires an NG tube for feeding. What should you
immediately do after inserting an NG tube for liquid enteral feedings?
A. Aspirate for gastric secretions with a syringe.
B. Begin feeding slowly to prevent cramping.
C. Get an X-ray of the tip of the tube within 24 hours.
D. Clamp off the tube until the feedings begin.
58. Stephanie, a 28 y.o. accident victim, requires TPN. The rationale for TPN is to provide:

A. Necessary fluids and electrolytes to the body.


B. Complete nutrition by the I.V. route.
C. Tube feedings for nutritional supplementation.
D. Dietary supplementation with liquid protein given between meals.
59. Type A chronic gastritis can be distinquished from type B by its ability to:

A. Cause atrophy of the parietal cells.


B. Affect only the antrum of the stomach.
C. Thin the lining of the stomach walls.
D. Decrease gastric secretions.
60. Matt is a 49 y.o. with a hiatal hernia that you are about to counsel. Health care counseling for
Matt should include which of the following instructions?

A. Restrict intake of high-carbohydrate foods.


B. Increase fluid intake with meals.
C. Increase fat intake.
D. Eat three regular meals a day.
61. Jerod is experiencing an acute episode of ulcerative colitis. Which is priority for this patient?

A. Replace lost fluid and sodium.


B. Monitor for increased serum glucose level from steroid therapy.
C. Restrict the dietary intake of foods high in potassium.
D. Note any change in the color and consistency of stools.
62. A 29 y.o. patient has an acute episode of ulcerative colitis. What diagnostic test confirms this
diagnosis?

A. Barium Swallow.
B. Stool examination.
C. Gastric analysis.
D. Sigmoidoscopy.
63. Eleanor, a 62 y.o. woman with diverticulosis is your patient. Which interventions would you
expect to include in her care?

A. Low-fiber diet and fluid restrictions.


B. Total parenteral nutrition and bed rest.
C. High-fiber diet and administration of psyllium.
D. Administration of analgesics and antacids.
64. Regina is a 46 y.o. woman with ulcerative colitis. You expect her stools to look like:

A. Watery and frothy.


B. Bloody and mucoid.
C. Firm and well-formed.
D. Alternating constipation and diarrhea.
65. Donald is a 61 y.o. man with diverticulitis. Diverticulitis is characterized by:

A. Periodic rectal hemorrhage.


B. Hypertension and tachycardia.
C. Vomiting and elevated temperature.
D. Crampy and lower left quadrant pain and low-grade fever.
66. Brenda, a 36 y.o. patient is on your floor with acute pancreatitis. Treatment for her includes:

A. Continuous peritoneal lavage.


B. Regular diet with increased fat.
C. Nutritional support with TPN.
D. Insertion of a T tube to drain the pancreas.
67. Glenda has cholelithiasis (gallstones). You expect her to complain of:

A. Pain in the right upper quadrant, radiating to the shoulder.


B. Pain in the right lower quadrant, with rebound tenderness.
C. Pain in the left upper quadrant, with shortness of breath.
D. Pain in the left lower quadrant, with mild cramping.
68. After an abdominal resection for colon cancer, Madeline returns to her room with a Jackson-
Pratt drain in place. The purpose of the drain is to:

A. Irrigate the incision with a saline solution.


B. Prevent bacterial infection of the incision.
C. Measure the amount of fluid lost after surgery.
D. Prevent accumulation of drainage in the wound.
69. Anthony, a 60 y.o. patient, has just undergone a bowel resection with a colostomy. During the
first 24 hours, which of the following observations about the stoma should you report to the doctor?

A. Pink color.
B. Light edema.
C. Small amount of oozing.
D. Trickles of bright red blood.
70. Your teaching Anthony how to use his new colostomy. How much skin should remain exposed
between the stoma and the ring of the appliance?

A. 1/16”
B. 1/4″
C. 1/2”
D. 1”
71. Claire, a 33 y.o. is on your floor with a possible bowel obstruction. Which intervention is
priority for her?

A. Obtain daily weights.


B. Measure abdominal girth.
C. Keep strict intake and output.
D. Encourage her to increase fluids.
72. Your patient has a GI tract that is functioning, but has the inability to swallow foods. Which is
the preferred method of feeding for your patient?

A. TPN
B. PPN
C. NG feeding
D. Oral liquid supplements
73. You’re patient is complaining of abdominal pain during assessment. What is your priority?

A. Auscultate to determine changes in bowel sounds.


B. Observe the contour of the abdomen.
C. Palpate the abdomen for a mass.
D. Percuss the abdomen to determine if fluid is present.
74. Before bowel surgery, Lee is to administer enemas until clear. During administration, he
complains of intestinal cramps. What do you do next?

A. Discontinue the procedure.


B. Lower the height of the enema container.
C. Complete the procedure as quickly as possible.
D. Continue administration of the enema as ordered without making any adjustments.
75. Leigh Ann is receiving pancrelipase (Viokase) for chronic pancreatitis. Which observation best
indicates the treatment is effective?

A. There is no skin breakdown.


B. Her appetite improves.
C. She loses more than 10 lbs.
D. Stools are less fatty and decreased in frequency.
76. Ralph has a history of alcohol abuse and has acute pancreatitis. Which lab value is most likely
to be elevated?

A. Calcium
B. Glucose
C. Magnesium
D. Potassium
77. Anna is 45 y.o. and has a bleeding ulcer. Despite multiple blood transfusions, her HGB is
7.5g/dl and HCT is 27%. Her doctor determines that surgical intervention is necessary and she
undergoes partial gastrectomy. Postoperative nursing care includes:

A. Giving pain medication Q6H.


B. Flushing the NG tube with sterile water.
C. Positioning her in high Fowler’s position.
D. Keeping her NPO until the return of peristalsis.
78. Sitty, a 66 y.o. patient underwent a colostomy for ruptured diverticulum. She did well during
the surgery and returned to your med-surg floor in stable condition. You assess her colostomy 2
days after surgery. Which finding do you report to the doctor?

A. Blanched stoma
B. Edematous stoma
C. Reddish-pink stoma
D. Brownish-black stoma
79. Sharon has cirrhosis of the liver and develops ascites. What intervention is necessary to
decrease the excessive accumulation of serous fluid in her peritoneal cavity?

A. Restrict fluids
B. Encourage ambulation
C. Increase sodium in the diet
D. Give antacids as prescribed
80. Katrina is diagnosed with lactose intolerance. To avoid complications with lack of calcium in
the diet, which food should be included in the diet?

A. Fruit
B. Whole grains
C. Milk and cheese products
D. Dark green, leafy vegetables
81. Nathaniel has severe pruritus due to having hepatitis B. What is the best intervention for his
comfort?

A. Give tepid baths.


B. Avoid lotions and creams.
C. Use hot water to increase vasodilation.
D. Use cold water to decrease the itching.
82. Rob is a 46 y.o. admitted to the hospital with a suspected diagnosis of Hepatitis B. He’s
jaundiced and reports weakness. Which intervention will you include in his care?

A. Regular exercise.
B. A low-protein diet.
C. Allow patient to select his meals.
D. Rest period after small, frequent meals.
83. You’re discharging Nathaniel with hepatitis B. Which statement suggests understanding by the
patient?

A. “Now I can never get hepatitis again.”


B. “I can safely give blood after 3 months.”
C. “I’ll never have a problem with my liver again, even if I drink alcohol.”
D. “My family knows that if I get tired and start vomiting, I may be getting sick again.”
84. Gail is scheduled for a cholecystectomy. After completion of preoperative teaching, Gail
states,”If I lie still and avoid turning after the operation, I’ll avoid pain. Do you think this is a good
idea?” What is the best response?
A. “You’ll need to turn from side to side every 2 hours.”
B. “It’s always a good idea to rest quietly after surgery.”
C. “The doctor will probably order you to lie flat for 24 hours.”
D. “Why don’t you decide about activity after you return from the recovery room?”
85. You’re caring for a 28 y.o. woman with hepatitis B. She’s concerned about the duration of her
recovery. Which response isn’t appropriate?

A. Encourage her to not worry about the future.


B. Encourage her to express her feelings about the illness.
C. Discuss the effects of hepatitis B on future health problems.
D. Provide avenues for financial counseling if she expresses the need.
86. Elmer is scheduled for a proctoscopy and has an I.V. The doctor wrote an order for 5mg of I.V.
diazepam(Valium). Which order is correct regarding diazepam?

A. Give diazepam in the I.V. port closest to the vein.


B. Mix diazepam with 50 ml of dextrose 5% in water and give over 15 minutes.
C. Give diazepam rapidly I.V. to prevent the bloodstream from diluting the drug mixture.
D. Question the order because I.V. administration of diazepam is contraindicated.
87. Annebell is being discharged with a colostomy, and you’re teaching her about colostomy care.
Which statement correctly describes a healthy stoma?

A. “At first, the stoma may bleed slightly when touched.”


B. “The stoma should appear dark and have a bluish hue.”
C. “A burning sensation under the stoma faceplate is normal.”
D. “The stoma should remain swollen away from the abdomen.”
88. A patient who underwent abdominal surgery now has a gaping incision due to delayed wound
healing. Which method is correct when you irrigate a gaping abdominal incision with sterile normal
saline solution, using a piston syringe?

A. Rapidly instill a stream of irrigating solution into the wound.


B. Apply a wet-to-dry dressing to the wound after the irrigation.
C. Moisten the area around the wound with normal saline solution after the irrigation.
D. Irrigate continuously until the solution becomes clear or all of the solution is used.
89. Hepatic encephalopathy develops when the blood level of which substance increases?

A. Ammonia
B. Amylase
C. Calcium
D. Potassium
90. Your patient recently had abdominal surgery and tells you that he feels a popping sensation in
his incision during a coughing spell, followed by severe pain. You anticipate an evisceration.
Which supplies should you take to his room?

A. A suture kit.
B. Sterile water and a suture kit.
C. Sterile water and sterile dressings.
D. Sterile saline solution and sterile dressings.
91. Findings during an endoscopic exam include a cobblestone appearance of the colon in your
patient. The findings are characteristic of which disorder?

A. Ulcer
B. Crohn’s disease
C. Chronic gastritis
D. Ulcerative colitis
92. What information is correct about stomach cancer?

A. Stomach pain is often a late symptom.


B. Surgery is often a successful treatment.
C. Chemotherapy and radiation are often successful treatments.
D. The patient can survive for an extended time with TPN.
93. Dark, tarry stools indicate bleeding in which location of the GI tract?

A. Upper colon.
B. Lower colon.
C. Upper GI tract.
D. Small intestine.
94. A patient has an acute upper GI hemorrhage. Your interventions include:

A. Treating hypovolemia.
B. Treating hypervolemia.
C. Controlling the bleeding source.
D. Treating shock and diagnosing the bleeding source.
95. You promote hemodynamic stability in a patient with upper GI bleeding by:

A. Encouraging oral fluid intake.


B. Monitoring central venous pressure.
C. Monitoring laboratory test results and vital signs.
D. Giving blood, electrolyte and fluid replacement.
96. You’re preparing a patient with a malignant tumor for colorectal surgery and subsequent
colostomy. The patient tells you he’s anxious. What should your initial step be in working with this
patient?

A. Determine what the patient already knows about colostomies.


B. Show the patient some pictures of colostomies.
C. Arrange for someone who has a colostomy to visit the patient.
D. Provide the patient with written material about colostomy care.
97. Your patient, Christopher, has a diagnosis of ulcerative colitis and has severe abdominal pain
aggravated by movement, rebound tenderness, fever, nausea, and decreased urine output. This may
indicate which complication?

A. Fistula.
B. Bowel perforation.
C. Bowel obstruction.
D. Abscess.
98. A patient has a severe exacerbation of ulcerative colitis. Long-term medications will probably
include:

A. Antacids.
B. Antibiotics.
C. Corticosteroids.
D. Histamine2-receptor blockers.
99. The student nurse is teaching the family of a patient with liver failure. You instruct them to
limit which foods in the patient’s diet?

A. Meats and beans.


B. Butter and gravies.
C. Potatoes and pastas.
D. Cakes and pastries.
100. An intubated patient is receiving continuous enteral feedings through a Salem sump tube at a
rate of 60ml/hr. Gastric residuals have been 30-40ml when monitored Q4H. You check the gastric
residual and aspirate 220ml. What is your first response to this finding?

A. Notify the doctor immediately.


B. Stop the feeding, and clamp the NG tube.
C. Discard the 220ml, and clamp the NG tube.
D. Give a prescribed GI stimulant such as metoclopramide (Reglan).
Answers and Rationales
1. Answer C. The normal serum amylase level is 25 to 151 units/L. With chronic cases of
pancreatitis, the rise in serum amylase levels usually does not exceed three times the
normal value. In acute pancreatitis, the value may exceed five times the normal value.
Options A and B are within normal limits. Option D is an extremely elevated level seen
in acute pancreatitis.
2. Answer C. Full liquid food items include items such as plain ice cream, sherbet,
breakfast drinks, milk, pudding and custard, soups that are strained, and strained
vegetable juices. A clear liquid diet consists of foods that are relatively transparent. The
food items in options A, B, and D are clear liquids.
3. Answer A. The client with cirrhosis needs to consume foods high in thiamine. Thiamine
is present in a variety of foods of plant and animal origin. Pork products are especially
rich in this vitamin. Other good food sources include nuts, whole grain cereals, and
legumes. Milk contains vitamins A, D, and B2. Poultry contains niacin. Broccoli
contains vitamins C, E, and K and folic acid
4. Answer A. Unless specifically indicated, residual amounts more than 100 mL require
holding the feeding. Therefore options B, C, and D are incorrect. Additionally, the
feeding is not discarded unless its contents are abnormal in color or characteristics.
5. Answer D. During the insertion of a nasogastric tube, if the client experiences difficulty
breathing or any respiratory distress, withdraw the tube slightly, stop the tube
advancement, and wait until the distress subsides. Options B and C are unnecessary.
Quickly inserting the tube is not an appropriate action because, in this situation, it may
be likely that the tube has entered the bronchus.
6. Answer A. If the nasogastric tube is in the stomach, the pH of the contents will be
acidic. Gastric aspirates have acidic pH values and should be 3.5 or lower. Option B
indicates a slightly acidic pH. Option C indicates a neutral pH. Option D indicates an
alkaline pH.
7. Answer C. When the nurse removes a nasogastric tube, the client is instructed to take
and hold a deep breath. This will close the epiglottis. This allows for easy withdrawal
through the esophagus into the nose. The nurse removes the tube with one smooth,
continuous pull.
8. Answer C. If a client has a nasogastric tube connected to suction, the nurse should wait
up to 30 minutes before reconnecting the tube to the suction apparatus to allow adequate
time for medication absorption. Aspirating the nasogastric tube will remove the
medication just administered. Low intermittent suction also will remove the medication
just administered. The client should not be placed in the supine position because of the
risk for aspiration.
9. Answer D. When the client has a Sengstaken-Blakemore tube, a pair of scissors must be
kept at the client’s bedside at all times. The client needs to be observed for sudden
respiratory distress, which occurs if the gastric balloon ruptures and the entire tube
moves upward. If this occurs, the nurse immediately cuts all balloon lumens and
removes the tube. An obturator and a Kelly clamp are kept at the bedside of a client with
a tracheostomy. An irrigation set may be kept at the bedside, but it is not the priority
item.
10.Answer A. Hepatitis A is transmitted by the fecal-oral route via contaminated food or
infected food handlers. Hepatitis B, C, and D are transmitted most commonly via
infected blood or body fluids.
11.Answer B. Laboratory indicators of hepatitis include elevated liver enzyme levels,
elevated serum bilirubin levels, elevated erythrocyte sedimentation rates, and
leukopenia. An elevated blood urea nitrogen level may indicate renal dysfunction. A
hemoglobin level is unrelated to this diagnosis.
12.Answer C. Meperidine (Demerol) rather than morphine sulfate is the medication of
choice to treat pain because morphine sulfate can cause spasms in the sphincter of Oddi.
Options A, B, and D are appropriate interventions for the client with acute pancreatitis.
13.Answer A. A barium swallow is an x-ray study that uses a substance called barium for
contrast to highlight abnormalities in the gastrointestinal tract. The client should fast for
8 to 12 hours before the test, depending on physician instructions. Most oral medications
also are withheld before the test. After the procedure, the nurse must monitor for
constipation, which can occur as a result of the presence of barium in the gastrointestinal
tract.
14.Answer C. The appropriate sequence for abdominal examination is inspection,
auscultation, percussion, and palpation. Auscultation is performed after inspection to
ensure that the motility of the bowel and bowel sounds are not altered by percussion or
palpation. Therefore, after inspecting the skin on the abdomen, the nurse should listen
for bowel sounds.
15.Answer D. The solution GoLYTELY is a bowel evacuant used to prepare a client for a
colonoscopy by cleansing the bowel. The solution is expected to cause a mild diarrhea
and will clear the bowel in 4 to 5 hours. Options A, B, and C are inappropriate actions.
16.Answer B. Chronic gastritis causes deterioration and atrophy of the lining of the
stomach, leading to the loss of the function of the parietal cells. The source of the
intrinsic factor is lost, which results in the inability to absorb vitamin B12. This leads to
the development of pernicious anemia. The client is not at risk for vitamin A, C, or E
deficiency.
17.Answer C. Indomethacin (Indocin) is a nonsteroidal anti-inflammatory drug and can
cause ulceration of the esophagus, stomach, or small intestine. Indomethacin is
contraindicated in a client with gastrointestinal disorders. Furosemide (Lasix) is a loop
diuretic. Digoxin is a cardiac medication. Propranolol (Inderal) is a β-adrenergic
blocker. Furosemide, digoxin, and propranolol are not contraindicated in clients with
gastric disorders.
18.Answer D. Following cholecystectomy, drainage from the T tube is initially bloody and
then turns to a greenish-brown color. The drainage is measured as output. The amount of
expected drainage will range from 500 to 1000 mL/day. The nurse would document the
output.
19.Answer D. Perforation of an ulcer is a surgical emergency and is characterized by
sudden, sharp, intolerable severe pain beginning in the midepigastric area and spreading
over the abdomen, which becomes rigid and board-like. Nausea and vomiting may
occur. Tachycardia may occur as hypovolemic shock develops. Numbness in the legs is
not an associated finding.
20.Answer C. A vagotomy, or cutting of the vagus nerve, is done to eliminate
parasympathetic stimulation of gastric secretion. Options A, B, and D are incorrect
descriptions of a vagotomy.
21.Answer C. In a Billroth II procedure, the proximal remnant of the stomach is
anastomosed to the proximal jejunum. Patency of the nasogastric tube is critical for
preventing the retention of gastric secretions. The nurse should never irrigate or
reposition the gastric tube after gastric surgery, unless specifically ordered by the
physician. In this situation, the nurse should clarify the order. Options A, B, and D are
appropriate postoperative interventions.
22.Answer C. Dumping syndrome is a term that refers to a constellation of vasomotor
symptoms that occurs after eating, especially following a Billroth II procedure. Early
manifestations usually occur within 30 minutes of eating and include vertigo,
tachycardia, syncope, sweating, pallor, palpitations, and the desire to lie down. The
nurse should instruct the client to decrease the amount of fluid taken at meals and to
avoid high-carbohydrate foods, including fluids such as fruit nectars; to assume a low-
Fowler’s position during meals; to lie down for 30 minutes after eating to delay gastric
emptying; and to take antispasmodics as prescribed.
23.Answer A. Early manifestations of dumping syndrome occur 5 to 30 minutes after
eating. Symptoms include vertigo, tachycardia, syncope, sweating, pallor, palpitations,
and the desire to lie down.
24.Answer B. Coughing is avoided following umbilical hernia repair to prevent disruption
of tissue integrity, which can occur because of the location of this surgical procedure.
Bed rest is not required following this surgical procedure. The client should take
analgesics as needed and as prescribed to control pain. A drain is not used in this
surgical procedure, although the client may be instructed in simple dressing changes.
25.Answer B. Following inguinal hernia repair, the client should be instructed to elevate the
scrotum and apply ice packs while in bed to decrease pain and swelling. The nurse also
should instruct the client to apply a scrotal support when out of bed. Heat will increase
swelling. Limiting oral fluids and a low-fiber diet can cause constipation.
26.Answer C. Rebound tenderness may indicate peritonitis. Bloody diarrhea is expected to
occur in ulcerative colitis. Because of the blood loss, the client may be hypotensive and
the hemoglobin level may be lower than normal. Signs of peritonitis must be reported to
the physician.
27.Answer B. Body image, disturbed relates to loss of bowel control, the presence of a
stoma, the release of fecal material onto the abdomen, the passage of flatus, odor, and
the need for an appliance (external pouch). No data in the question support options A
and C. Nutrition: less than body requirements, imbalanced is the more likely nursing
diagnosis.
28.Answer A. Crohn’s disease is characterized by nonbloody diarrhea of usually not more
than four to five stools daily. Over time, the diarrhea episodes increase in frequency,
duration, and severity. Options B, C, and D are not characteristics of Crohn’s disease.
29.Answer B. If cramping occurs during a colostomy irrigation, the irrigation flow is
stopped temporarily and the client is allowed to rest. Cramping may occur from an
infusion that is too rapid or is causing too much pressure. The physician does not need to
be notified. Increasing the height of the irrigation will cause further discomfort.
Medicating the client for pain is not the appropriate action in this situation.
30.Answer A. To enhance effectiveness of the irrigation and fecal returns, the client is
instructed to increase fluid intake and to take other measures to prevent constipation.
Options B, C and D will not enhance the effectiveness of this procedure.
31.Answer A. Blood pressure decreases as the body is unable to maintain normal oncotic
pressure with liver failure, so patients with liver failure require close blood pressure
monitoring. Increased capillary permeability, abnormal peripheral vasodilation, and
excess rennin released from the kidney’s aren’t direct ramifications of liver failure.
32.Answer B. Good circulation causes tissues to be moist and red, so a healthy, well-healed
stoma appears red and moist.
33.Answer A. A colostomy in the sigmoid colon produces a solid, formed stool.
34.Answer D. High-fiber foods stimulate peristalsis, and a result, flatus. Yogurt reduces gas
formation.
35.Answer B. A proper fit protects the skin, but doesn’t impair circulation. A 1/16” should
be cut.
36.Answer B. Observation, auscultation, percussion, palpation
37.Answer D. An ileoanal reservoir is created in two stages. The two surgeries are about 2
to 3 months apart. First, diseased intestines are removed and a temporary loop ileostomy
is created. Second, the loop ileostomy is closed and stool goes to the reservoir and out
through the anus.
38.Answer C. The large intestine absorbs large amounts of water so the initial output from
the ileostomy may be as much as 1500 to 2000 ml/24 hours. Gradually, the small
intestine absorbs more fluid and the output decreases.
39.Answer B. To avoid overloading the small intestine, encourage the patient to eat six
small, regularly spaced meals.
40.Answer B. Making observations about what you see or hear is a useful therapeutic
technique. This way, you acknowledge that you are interested in what the patient is
saying and feeling.
41.Answer C. After a Billroth II procedure, a large amount of hypertonic fluid enters the
intestine. This causes extracellular fluid to move rapidly into the bowel, reducing
circulating blood volume and producing vasomotor symptoms. Vasomotor symptoms
produced by dumping syndrome include dizziness and sweating, tachycardia, syncope,
pallor, and palpitations.
42.Answer A. Gastric emptying time can be delayed by omitting fluids from your patient’s
meal. A diet low in carbs and high in fat & protein is recommended to treat dumping
syndrome.
43.Answer B. Ascites puts pressure on the diaphragm. Paracentesis is done to remove fluid
and reducing pressure on the diaphragm. The goal is to improve the patient’s breathing.
The others are signs of cirrhosis that aren’t relieved by paracentesis.
44.Answer A. A full bladder can interfere with paracentesis and be punctured inadvertently.
45.Answer B. Cover the organs with a sterile, nonadherent dressing moistened with normal
saline. Do this to prevent infection and to keep the organs from drying out.
46.Answer A. Asterixis is an early neurologic sign of hepatic encephalopathy elicited by
asking the patient to hold her arms stretched out. Asterixis is present if the hands rapidly
extend and flex.
47.Answer A. You may administer the laxative lactulose to reduce ammonia levels in the
colon.
48.Answer A. Achalasia is characterized by incomplete relaxation of the LES, dilation of
the lower esophagus, and a lack of esophageal peristalsis. Because nitrates relax the
lower esophageal sphincter, expect to give Isordil orally or sublingually.
49.Answer C. Eating in the upright position aids in emptying the esophagus. Doing the
opposite of the other three also may be helpful.
50.Answer C. Pancreatitis involves activation of pancreatic enzymes, such as amylase and
lipase. These levels are elevated in a patient with acute pancreatitis.
51.Answer D. The normal range of specific gravity of urine is 1.010 to 1.025; a value of
1.030 may be seen with dehydration.
52.Answer C. Teach the pt to avoid activities that increase intra-abdominal pressure such as
coughing, sneezing, or straining with a bowel movement.
53.Answer C. Because obesity weakens the abdominal muscles, advise weight loss for the
patient who has had a hernia repair.
54.Answer B. After a liver biopsy, the patient is placed on the right side to compress the
liver and to reduce the risk of bleeding or bile leakage.
55.Answer A. Signs and Symptoms of pneumothorax include dyspnea and decreased or
absent breath sounds over the affected lung (right lung).
56.Answer A. An NG tube is inserted into the patients stomach to drain fluid and gas.
57.Answer A. Aspirating the stomach contents confirms correct placement. If an X-ray is
ordered, it should be done immediately, not in 24 hours.
58.Answer B. TPN is given I.V. to provide all the nutrients your patient needs. TPN isn’t a
tube feeding nor is it a liquid dietary supplement.
59.Answer A. Type A causes changes in parietal cells.
60.Answer B. Increasing fluids helps empty the stomach. A high carb diet isn’t restricted
and fat intake shouldn’t be increased.
61.Answer A. Diarrhea d/t an acute episode of ulcerative colitis leads to fluid & electrolyte
losses so fluid replacement takes priority.
62.Answer D. Sigmoidoscopy allows direct observation of the colon mucosa for changes,
and if needed, biopsy.
63.Answer C. She needs a high-fiber diet and a psyllium (bulk laxative) to promote normal
soft stools.
64.Answer B. Stools from ulcerative colitis are often bloody and contain mucus.
65.Answer D. One sign of acute diverticulitis is crampy lower left quadrant pain. A low-
grade fever is another common sign.
66.Answer C. With acute pancreatitis, you need to rest the GI tract by TPN as nutritional
support.
67.Answer A. The gallbladder is located in the RUQ and a frequent sign of gallstones is
pain radiating to the shoulder.
68.Answer D. A Jackson-Pratt drain promotes wound healing by allowing fluid to escape
from the wound.
69.Answer D. After creation of a colostomy, expect to see a stoma that is pink, slightly
edematous, with some oozing. Bright red blood, regardless of amount, indicates bleeding
and should be reported to the doctor.
70.Answer A. Only a small amount of skin should be exposed and more than 1/16” of skin
allows the excretement to irritate the skin.
71.Answer B. Measuring abdominal girth provides quantitative information about increases
or decreases in the amount of distention.
72.Answer C. Because the GI tract is functioning, feeding methods involve the enteral route
which bypasses the mouth but allows for a major portion of the GI tract to be used.
73.Answer B. The first step in assessing the abdomen is to observe its shape and contour,
then auscultate, palpate, and then percuss.
74.Answer B. Lowering the height decreases the amount of flow, allowing him to tolerate
more fluid.
75.Answer D. Pancrelipase provides the exocrine pancreatic enzyme necessary for proper
protein, fat, and carb digestion. With increased fat digestion and absorption, stools
become less frequent and normal in appearance.
76.Answer B. Glucose level increases and diabetes mellitus may result d/t the pancreatic
damage to the islets of langerhans.
77.Answer D. After surgery, she remains NPO until peristaltic activity returns. This
decreases the risk for abdominal distention and obstruction.
78.Answer D. A brownish-black color indicates lack of blood flow, and maybe necrosis.
79.Answer A. Restricting fluids decrease the amount of body fluid and the accumulation of
fluid in the peritoneal space.
80.Answer D. Dark green, leafy vegetables are rich in calcium.
81.Answer A. For pruritus, care should include tepid sponge baths and use of emollient
creams and lotions.
82.Answer D. Rest periods and small frequent meals is indicated during the acute phase of
hepatitis B.
83.Answer D. Hepatitis B can recur. Patients who have had hepatitis are permanently
barred from donating blood. Alcohol is metabolized by the liver and should be avoided
by those who have or had hepatitis B.
84.Answer A. To prevent venous stasis and improve muscle tone, circulation, and
respiratory function, encourage her to move after surgery.
85.Answer A. Telling her not to worry minimizes her feelings.
86.Answer A. Diazepam is absorbed by the plastic I.V. tubing and should be given in the
port closest to the vein.
87.Answer A. For the first few days to a week, slight bleeding normally occurs when the
stoma is touched because the surgical site is still new. She should report profuse
bleeding immediately.
88.Answer D. To wash away tissue debris and drainage effectively, irrigate the wound until
the solution becomes clear or all the solution is used.
89.Answer A. Ammonia levels increase d/t improper shunting of blood, causing ammonia
to enter systemic circulation, which carries it to the brain.
90.Answer D. Saline solution is isotonic, or close to body fluids in content, and is used
along with sterile dressings to cover an eviscerated wound and keep it moist.
91.Answer B. Crohn’s disease penetrates the mucosa of the colon through all layers and
destroys the colon in patches, which creates a cobblestone appearance.
92.Answer A. Stomach pain is often a late sign of stomach cancer; outcomes are
particularly poor when the cancer reaches that point. Surgery, chemotherapy, and
radiation have minimal positive effects. TPN may enhance the growth of the cancer.
93.Answer C. Melena is the passage of dark, tarry stools that contain a large amount of
digested blood. It occurs with bleeding from the upper GI tract.
94.Answer A. A patient with an acute upper GI hemorrhage must be treated for
hypovolemia and hemorrhagic shock. You as a nurse can’t diagnose the problem.
Controlling the bleeding may require surgery or intensive medical treatment.
95.Answer D. To stabilize a patient with acute bleeding, NS or LR solution is given I.V.
until BP rises and urine output returns to 30ml/hr.
96.Answer A. Initially, you should assess the patient’s knowledge about colostomies and
how it will affect his lifestyle.
97.Answer B. An inflammatory condition that affects the surface of the colon, ulcerative
colitis causes friability and erosions with bleeding. Patients with ulcerative colitis are at
increased risk for bowel perforation, toxic megacolon, hemorrhage, cancer, and other
anorectal and systemic complications.
98.Answer C. Medications to control inflammation such as corticosteroids are used for
long-term treatment.
99.Answer A. Meats and beans are high-protein foods. In liver failure, the liver is unable to
metabolize protein adequately, causing protein by-products to build up in the body rather
than be excreted.
100. Answer B. A gastric residual greater than 2 hours worth of feeding or 100-150ml is
considered too high. The feeding should be stopped; NG tube clamped, and then allow
time for the stomach to empty before additional feeding is added.
1. During preparation for bowel surgery, a male client receives an antibiotic to reduce intestinal
bacteria. Antibiotic therapy may interfere with synthesis of which vitamin and may lead to
hypoprothrombinemia?

A. vitamin A
B. vitamin D
C. vitamin E
D. vitamin K
2. When evaluating a male client for complications of acute pancreatitis, the nurse would observe
for:

A. increased intracranial pressure.


B. decreased urine output.
C. bradycardia.
D. hypertension.
3. A male client with a recent history of rectal bleeding is being prepared for a colonoscopy. How
should the nurse position the client for this test initially?

A. Lying on the right side with legs straight


B. Lying on the left side with knees bent
C. Prone with the torso elevated
D. Bent over with hands touching the floor
4. A male client with extreme weakness, pallor, weak peripheral pulses, and disorientation is
admitted to the emergency department. His wife reports that he has been “spitting up blood.” A
Mallory-Weiss tear is suspected, and the nurse begins taking a client history from the client’s wife.
The question by the nurse that demonstrates her understanding of Mallory-Weiss tearing is:

A. “Tell me about your husband’s alcohol usage.”


B. “Is your husband being treated for tuberculosis?”
C. “Has your husband recently fallen or injured his chest?”
D. “Describe spices and condiments your husband uses on food.”
5. Which of the following nursing interventions should the nurse perform for a female client
receiving enteral feedings through a gastrostomy tube?

A. Change the tube feeding solutions and tubing at least every 24 hours.
B. Maintain the head of the bed at a 15-degree elevation continuously.
C. Check the gastrostomy tube for position every 2 days.
D. Maintain the client on bed rest during the feedings.
6. A male client is recovering from a small-bowel resection. To relieve pain, the physician
prescribes meperidine (Demerol), 75 mg I.M. every 4 hours. How soon after administration should
meperidine’s onset of action occur?

A. 5 to 10 minutes
B. 15 to 30 minutes
C. 30 to 60 minutes
D. 2 to 4 hours
7. The nurse is caring for a male client with cirrhosis. Which assessment findings indicate that the
client has deficient vitamin K absorption caused by this hepatic disease?

A. Dyspnea and fatigue


B. Ascites and orthopnea
C. Purpura and petechiae
D. Gynecomastia and testicular atrophy
8. Which condition is most likely to have a nursing diagnosis of fluid volume deficit?

A. Appendicitis
B. Pancreatitis
C. Cholecystitis
D. Gastric ulcer
9. While a female client is being prepared for discharge, the nasogastric (NG) feeding tube
becomes clogged. To remedy this problem and teach the client’s family how to deal with it at
home, what should the nurse do?

A. Irrigate the tube with cola.


B. Advance the tube into the intestine.
C. Apply intermittent suction to the tube.
D. Withdraw the obstruction with a 30-ml syringe.
10. A male client with pancreatitis complains of pain. The nurse expects the physician to prescribe
meperidine (Demerol) instead of morphine to relieve pain because:

A. meperidine provides a better, more prolonged analgesic effect.


B. morphine may cause spasms of Oddi’s sphincter.
C. meperidine is less addictive than morphine.
D. morphine may cause hepatic dysfunction.
11. Mandy, an adolescent girl is admitted to an acute care facility with severe malnutrition. After a
thorough examination, the physician diagnoses anorexia nervosa. When developing the plan of care
for this client, the nurse is most likely to include which nursing diagnosis?

A. Hopelessness
B. Powerlessness
C. Chronic low self esteem
D. Deficient knowledge
12. Which diagnostic test would be used first to evaluate a client with upper GI bleeding?

A. Endoscopy
B. Upper GI series
C. Hemoglobin (Hb) levels and hematocrit (HCT)
D. Arteriography
13. A female client who has just been diagnosed with hepatitis A asks, “How could I have gotten
this disease?” What is the nurse’s best response?

A. “You may have eaten contaminated restaurant food.”


B. “You could have gotten it by using I.V. drugs.”
C. “You must have received an infected blood transfusion.”
D. “You probably got it by engaging in unprotected sex.”
14. When preparing a male client, age 51, for surgery to treat appendicitis, the nurse formulates a
nursing diagnosis of Risk for infection related to inflammation, perforation, and surgery. What is
the rationale for choosing this nursing diagnosis?

A. Obstruction of the appendix may increase venous drainage and cause the appendix to
rupture.
B. Obstruction of the appendix reduces arterial flow, leading to ischemia, inflammation,
and rupture of the appendix.
C. The appendix may develop gangrene and rupture, especially in a middle-aged client.
D. Infection of the appendix diminishes necrotic arterial blood flow and increases venous
drainage.
15. A female client with hepatitis C develops liver failure and GI hemorrhage. The blood products
that would most likely bring about hemostasis in the client are:

A. whole blood and albumin.


B. platelets and packed red blood cells.
C. fresh frozen plasma and whole blood.
D. cryoprecipitate and fresh frozen plasma.
16. To prevent gastroesophageal reflux in a male client with hiatal hernia, the nurse should provide
which discharge instruction?

A. “Lie down after meals to promote digestion.”


B. “Avoid coffee and alcoholic beverages.”
C. “Take antacids with meals.”
D. “Limit fluid intake with meals.”
17. The nurse caring for a client with small-bowel obstruction would plan to implement which
nursing intervention first?

A. Administering pain medication


B. Obtaining a blood sample for laboratory studies
C. Preparing to insert a nasogastric (NG) tube
D. Administering I.V. fluids
18. A female client with dysphagia is being prepared for discharge. Which outcome indicates that
the client is ready for discharge?

A. The client doesn’t exhibit rectal tenesmus.


B. The client is free from esophagitis and achalasia.
C. The client reports diminished duodenal inflammation.
D. The client has normal gastric structures.
19. A male client undergoes total gastrectomy. Several hours after surgery, the nurse notes that the
client’s nasogastric (NG) tube has stopped draining. How should the nurse respond?

A. Notify the physician


B. Reposition the tube
C. Irrigate the tube
D. Increase the suction level
20. What laboratory finding is the primary diagnostic indicator for pancreatitis?

A. Elevated blood urea nitrogen (BUN)


B. Elevated serum lipase
C. Elevated aspartate aminotransferase (AST)
D. Increased lactate dehydrogenase (LD)
21. A male client with cholelithiasis has a gallstone lodged in the common bile duct. When
assessing this client, the nurse expects to note:

A. yellow sclerae.
B. light amber urine.
C. circumoral pallor.
D. black, tarry stools.
22. Nurse Hannah is teaching a group of middle-aged men about peptic ulcers. When discussing
risk factors for peptic ulcers, the nurse should mention:

A. a sedentary lifestyle and smoking.


B. a history of hemorrhoids and smoking.
C. alcohol abuse and a history of acute renal failure.
D. alcohol abuse and smoking.
23. While palpating a female client’s right upper quadrant (RUQ), the nurse would expect to find
which of the following structures?

A. Sigmoid colon
B. Appendix
C. Spleen
D. Liver
24. A male client has undergone a colon resection. While turning him, wound dehiscence with
evisceration occurs. The nurse’s first response is to:

A. call the physician.


B. place saline-soaked sterile dressings on the wound.
C. take a blood pressure and pulse.
D. pull the dehiscence closed.
25. The nurse is monitoring a female client receiving paregoric to treat diarrhea for drug
interactions. Which drugs can produce additive constipation when given with an opium
preparation?

A. Antiarrhythmic drugs
B. Anticholinergic drugs
C. Anticoagulant drugs
D. Antihypertensive drugs
26. A male client is recovering from an ileostomy that was performed to treat inflammatory bowel
disease. During discharge teaching, the nurse should stress the importance of:

A. increasing fluid intake to prevent dehydration.


B. wearing an appliance pouch only at bedtime.
C. consuming a low-protein, high-fiber diet.
D. taking only enteric-coated medications.
27. The nurse is caring for a female client with active upper GI bleeding. What is the appropriate
diet for this client during the first 24 hours after admission?

A. Regular diet
B. Skim milk
C. Nothing by mouth
D. Clear liquids
28. A male client has just been diagnosed with hepatitis A. On assessment, the nurse expects to
note:

A. severe abdominal pain radiating to the shoulder.


B. anorexia, nausea, and vomiting.
C. eructation and constipation.
D. abdominal ascites.
29. A female client with viral hepatitis A is being treated in an acute care facility. Because the
client requires enteric precautions, the nurse should:

A. place the client in a private room.


B. wear a mask when handling the client’s bedpan.
C. wash the hands after touching the client.
D. wear a gown when providing personal care for the client.
30. Which of the following factors can cause hepatitis A?

A. Contact with infected blood


B. Blood transfusions with infected blood
C. Eating contaminated shellfish
D. Sexual contact with an infected person
31. Your patient with peritonitis is NPO and complaining of thirst. What is your priority?

A. Increase the I.V. infusion rate.


B. Use diversion activities.
C. Provide frequent mouth care.
D. Give ice chips every 15 minutes.
32. Kevin has a history of peptic ulcer disease and vomits coffee-ground emesis. What does this
indicate?

A. He has fresh, active upper GI bleeding.


B. He needs immediate saline gastric lavage.
C. His gastric bleeding occurred 2 hours earlier.
D. He needs a transfusion of packed RBC’s.
33. A 53 y.o. patient has undergone a partial gastrectomy for adenocarcinoma of the stomach. An
NG tube is in place and is connected to low continuous suction. During the immediate
postoperative period, you expect the gastric secretions to be which color?

A. Brown.
B. Clear.
C. Red.
D. Yellow.
34. Your patient has a retractable gastric peptic ulcer and has had a gastric vagotomy. Which factor
increases as a result of vagotomy?

A. Peristalsis.
B. Gastric acidity.
C. Gastric motility.
D. Gastric pH.
35. Christina is receiving an enteral feeding that requires a concentration of 80ml of supplement
mixed with 20 ml of water. How much water do you mix with an 8 oz (240ml) can of feeding?

A. 60 ml.
B. 70 ml.
C. 80 ml.
D. 90 ml.
36. Which stoma would you expect a malodorous, enzyme-rich, caustic liquid output that is
yellow, green, or brown?

A. Ileostomy.
B. Ascending colostomy.
C. Transverse colostomy.
D. Descending colostomy.
37. George has a T tube in place after gallbladder surgery. Before discharge, what information or
instructions should be given regarding the T tube drainage?

A. “If there is any drainage, notify the surgeon immediately.”


B. “The drainage will decrease daily until the bile duct heals.”
C. “First, the drainage is dark green; then it becomes dark yellow.”
D. “If the drainage stops, milk the tube toward the puncture wound.”
38. Your patient Maria takes NSAIDS for her degenerative joint disease, has developed peptic
ulcer disease. Which drug is useful in preventing NSAID-induced peptic ulcer disease?

A. Calcium carbonate (Tums)


B. Famotidine (Pepcid)
C. Misoprostol (Cytotec)
D. Sucralfate (Carafate)
39. The student nurse is participating in colorectal cancer-screening program. Which patient has
the fewest risk factors for colon cancer?
A. Janice, a 45 y.o. with a 25-year history of ulcerative colitis
B. George, a 50 y.o. whose father died of colon cancer
C. Herman, a 60 y.o. who follows a low-fat, high-fiber diet
D. Sissy, a 72 y.o. with a history of breast cancer
40. You’re patient, post-op drainage of a pelvic abscess secondary to diverticulitis, begins to cough
violently after drinking water. His wound has ruptured and a small segment of the bowel is
protruding. What’s your priority?

A. Ask the patient what happened, call the doctor, and cover the area with a water-soaked
bedsheet.
B. Obtain vital signs, call the doctor, and obtain emergency orders.
C. Have a CAN hold the wound together while you obtain vital signs, call the doctor and
flex the patient’s knees.
D. Have the doctor called while you remain with the patient, flex the patient’s knees, and
cover the wound with sterile towels soaked in sterile saline solution.

Answers and Rationales


1. Answer D. Intestinal bacteria synthesize such nutritional substances as vitamin K,
thiamine, riboflavin, vitamin B12, folic acid, biotin, and nicotinic acid. Therefore,
antibiotic therapy may interfere with synthesis of these substances, including vitamin K.
Intestinal bacteria don’t synthesize vitamins A, D, or E.

2. Answer B. Acute pancreatitis can cause decreased urine output, which results from the
renal failure that sometimes accompanies this condition. Intracranial pressure neither
increases nor decreases in a client with pancreatitis. Tachycardia, not bradycardia,
usually is associated with pulmonary or hypovolemic complications of pancreatitis.
Hypotension can be caused by a hypovolemic complication, but hypertension usually
isn’t related to acute pancreatitis.

3. Answer B. For a colonoscopy, the nurse initially should position the client on the left
side with knees bent. Placing the client on the right side with legs straight, prone with
the torso elevated, or bent over with hands touching the floor wouldn’t allow proper
visualization of the large intestine.

4. Answer A. A Mallory-Weiss tear is associated with massive bleeding after a tear occurs
in the mucous membrane at the junction of the esophagus and stomach. There is a strong
relationship between ethanol usage, resultant vomiting, and a Mallory-Weiss tear. The
bleeding is coming from the stomach, not from the lungs as would be true in some cases
of tuberculosis. A Mallory-Weiss tear doesn’t occur from chest injuries or falls and isn’t
associated with eating spicy foods.

5. Answer A. Tube feeding solutions and tubing should be changed every 24 hours, or
more frequently if the feeding requires it. Doing so prevents contamination and bacterial
growth. The head of the bed should be elevated 30 to 45 degrees continuously to prevent
aspiration. Checking for gastrostomy tube placement is performed before initiating the
feedings and every 4 hours during continuous feedings. Clients may ambulate during
feedings.

6. Answer B. Meperidine’s onset of action is 15 to 30 minutes. It peaks between 30 and 60


minutes and has a duration of action of 2 to 4 hours.

7. Answer C. A hepatic disorder, such as cirrhosis, may disrupt the liver’s normal use of
vitamin K to produce prothrombin (a clotting factor). Consequently, the nurse should
monitor the client for signs of bleeding, including purpura and petechiae. Dyspnea and
fatigue suggest anemia. Ascites and orthopnea are unrelated to vitamin K absorption.
Gynecomastia and testicular atrophy result from decreased estrogen metabolism by the
diseased liver.

8. Answer B. Hypovolemic shock from fluid shifts is a major factor in acute pancreatitis.
The other conditions are less likely to exhibit fluid volume deficit.

9. Answer A. The nurse should irrigate the tube with cola because its effervescence and
acidity are suited to the purpose, it’s inexpensive, and it’s readily available in most
homes. Advancing the NG tube is inappropriate because the tube is designed to stay in
the stomach and isn’t long enough to reach the intestines. Applying intermittent suction
or using a syringe for aspiration is unlikely to dislodge the material clogging the tube but
may create excess pressure. Intermittent suction may even collapse the tube.

10.Answer B. For a client with pancreatitis, the physician will probably avoid prescribing
morphine because this drug may trigger spasms of the sphincter of Oddi (a sphincter at
the end of the pancreatic duct), causing irritation of the pancreas. Meperidine has a
somewhat shorter duration of action than morphine. The two drugs are equally addictive.
Morphine isn’t associated with hepatic dysfunction.

11.Answer C. Young women with Chronic low self esteem — are at highest risk for
anorexia nervosa because they perceive being thin as a way to improve their self-
confidence. Hopelessness and Powerlessness are inappropriate nursing diagnoses
because clients with anorexia nervosa seldom feel hopeless or powerless; instead, they
use food to control their desire to be thin and hope that restricting food intake will
achieve this goal. Anorexia nervosa doesn’t result from a knowledge deficit, such as one
regarding good nutrition.

12.Answer A. Endoscopy permits direct evaluation of the upper GI tract and can detect
90% of bleeding lesions. An upper GI series, or barium study, usually isn’t the
diagnostic method of choice, especially in a client with acute active bleeding who’s
vomiting and unstable. An upper GI series is also less accurate than endoscopy.
Although an upper GI series might confirm the presence of a lesion, it wouldn’t
necessarily reveal whether the lesion is bleeding. Hb levels and HCT, which indicate
loss of blood volume, aren’t always reliable indicators of GI bleeding because a decrease
in these values may not be seen for several hours. Arteriography is an invasive study
associated with life-threatening complications and wouldn’t be used for an initial
evaluation.

13.Answer A. Hepatitis A virus typically is transmitted by the oral-fecal route —


commonly by consuming food contaminated by infected food handlers. The virus isn’t
transmitted by the I.V. route, blood transfusions, or unprotected sex. Hepatitis B can be
transmitted by I.V. drug use or blood transfusion. Hepatitis C can be transmitted by
unprotected sex.

14.Answer B. A client with appendicitis is at risk for infection related to inflammation,


perforation, and surgery because obstruction of the appendix causes mucus fluid to build
up, increasing pressure in the appendix and compressing venous outflow drainage. The
pressure continues to rise with venous obstruction; arterial blood flow then decreases,
leading to ischemia from lack of perfusion. Inflammation and bacterial growth follow,
and swelling continues to raise pressure within the appendix, resulting in gangrene and
rupture. Geriatric, not middle-aged, clients are especially susceptible to appendix
rupture.

15.Answer D. The liver is vital in the synthesis of clotting factors, so when it’s diseased or
dysfunctional, as in hepatitis C, bleeding occurs. Treatment consists of administering
blood products that aid clotting. These include fresh frozen plasma containing fibrinogen
and cryoprecipitate, which have most of the clotting factors. Although administering
whole blood, albumin, and packed cells will contribute to hemostasis, those products
aren’t specifically used to treat hemostasis. Platelets are helpful, but the best answer is
cryoprecipitate and fresh frozen plasma.

16.Answer B. To prevent reflux of stomach acid into the esophagus, the nurse should
advise the client to avoid foods and beverages that increase stomach acid, such as coffee
and alcohol. The nurse also should teach the client to avoid lying down after meals,
which can aggravate reflux, and to take antacids after eating. The client need not limit
fluid intake with meals as long as the fluids aren’t gastric irritants.

17.Answer D. I.V. infusions containing normal saline solution and potassium should be
given first to maintain fluid and electrolyte balance. For the client’s comfort and to assist
in bowel decompression, the nurse should prepare to insert an NG tube next. A blood
sample is then obtained for laboratory studies to aid in the diagnosis of bowel
obstruction and guide treatment. Blood studies usually include a complete blood count,
serum electrolyte levels, and blood urea nitrogen level. Pain medication often is
withheld until obstruction is diagnosed because analgesics can decrease intestinal
motility.

18.Answer B. Dysphagia may be the reason why a client with esophagitis or achalasia seeks
treatment. Dysphagia isn’t associated with rectal tenesmus, duodenal inflammation, or
abnormal gastric structures.

19.Answer A. An NG tube that fails to drain during the postoperative period should be
reported to the physician immediately. It may be clogged, which could increase pressure
on the suture site because fluid isn’t draining adequately. Repositioning or irrigating an
NG tube in a client who has undergone gastric surgery can disrupt the anastomosis.
Increasing the level of suction may cause trauma to GI mucosa or the suture line.

20.Answer B. Elevation of serum lipase is the most reliable indicator of pancreatitis


because this enzyme is produced solely by the pancreas. A client’s BUN is typically
elevated in relation to renal dysfunction; the AST, in relation to liver dysfunction; and
LD, in relation to damaged cardiac muscle.

21.Answer A. Yellow sclerae may be the first sign of jaundice, which occurs when the
common bile duct is obstructed. Urine normally is light amber. Circumoral pallor and
black, tarry stools don’t occur in common bile duct obstruction; they are signs of
hypoxia and GI bleeding, respectively.

22.Answer D. Risk factors for peptic (gastric and duodenal) ulcers include alcohol abuse,
smoking, and stress. A sedentary lifestyle and a history of hemorrhoids aren’t risk
factors for peptic ulcers. Chronic renal failure, not acute renal failure, is associated with
duodenal ulcers.

23.Answer D. The RUQ contains the liver, gallbladder, duodenum, head of the pancreas,
hepatic flexure of the colon, portions of the ascending and transverse colon, and a
portion of the right kidney. The sigmoid colon is located in the left lower quadrant; the
appendix, in the right lower quadrant; and the spleen, in the left upper quadrant.

24.Answer B. The nurse should first place saline-soaked sterile dressings on the open
wound to prevent tissue drying and possible infection. Then the nurse should call the
physician and take the client’s vital signs. The dehiscence needs to be surgically closed,
so the nurse should never try to close it.

25.Answer B. Paregoric has an additive effect of constipation when used with


anticholinergic drugs. Antiarrhythmics, anticoagulants, and antihypertensives aren’t
known to interact with paregoric.

26.Answer A. Because stool forms in the large intestine, an ileostomy typically drains
liquid waste. To avoid fluid loss through ileostomy drainage, the nurse should instruct
the client to increase fluid intake. The nurse should teach the client to wear a collection
appliance at all times because ileostomy drainage is incontinent, to avoid high-fiber
foods because they may irritate the intestines, and to avoid enteric-coated medications
because the body can’t absorb them after an ileostomy

27.Answer C. Shock and bleeding must be controlled before oral intake, so the client
should receive nothing by mouth. A regular diet is incorrect. When the bleeding is
controlled, the diet is gradually increased, starting with ice chips and then clear liquids.
Skim milk shouldn’t be given because it increases gastric acid production, which could
prolong bleeding. A liquid diet is the first diet offered after bleeding and shock are
controlled.
28.Answer B. Hallmark signs and symptoms of hepatitis A include anorexia, nausea,
vomiting, fatigue, and weakness. Abdominal pain may occur but doesn’t radiate to the
shoulder. Eructation and constipation are common in gallbladder disease, not hepatitis
A. Abdominal ascites is a sign of advanced hepatic disease, not an early sign of hepatitis
A.

29.Answer C. To maintain enteric precautions, the nurse must wash the hands after
touching the client or potentially contaminated articles and before caring for another
client. A private room is warranted only if the client has poor hygiene — for instance, if
the client is unlikely to wash the hands after touching infective material or is likely to
share contaminated articles with other clients. For enteric precautions, the nurse need not
wear a mask and must wear a gown only if soiling from fecal matter is likely.

30.Answer C. Hepatitis A can be caused by consuming contaminated water, milk, or food


— especially shellfish from contaminated water. Hepatitis B is caused by blood and
sexual contact with an infected person. Hepatitis C is usually caused by contact with
infected blood, including receiving blood transfusions.
31.Answer C. Frequent mouth care helps relieve dry mouth.
32.Answer C. Coffee-ground emesis occurs when there is upper GI bleeding that has
undergone gastric digestion. For blood to appear as coffee-ground emesis, it would have
to be digested for approximately 2 hours.
33.Answer C. Normally, drainage is bloody for the first 24 hours after a partial
gastrectomy; then it changes to brown-tinged and then to yellow or clear.
34.Answer D. If the vagus nerve is cut as it enters the stomach, gastric acid secretion is
decreased, but intestinal motility is also decreased and gastric emptying is delayed.
Because gastric acids are decreased, gastric pH increases.
35.Answer A. Dosage problem. It’s 80/20 = 240/X. X=60.
36.Answer A. The output from an Ileostomy is described.
37.Answer B. As healing occurs from the bile duct, bile drains from the tube; the amount of
bile should decrease. Teach the patient to expect dark green drainage and to notify the
doctor if drainage stops.
38.Answer C. Misoprostol restores prostaglandins that protect the stomach from NSAIDS,
which diminish the prostaglandins.
39.Answer C.
40.Answer D.
1. Which of the following complications is thought to be the most common cause of appendicitis?

A. A fecalith
B. Bowel kinking
C. Internal bowel occlusion
D. Abdominal bowel swelling
2. Which of the following terms best describes the pain associated with appendicitis?

A. Aching
B. Fleeting
C. Intermittent
D. Steady
3. Which of the following nursing interventions should be implemented to manage a client with
appendicitis?

A. Assessing for pain


B. Encouraging oral intake of clear fluids
C. Providing discharge teaching
D. Assessing for symptoms of peritonitis
4. Which of the following definitions best describes gastritis?

A. Erosion of the gastric mucosa


B. Inflammation of a diverticulum
C. Inflammation of the gastric mucosa
D. Reflux of stomach acid into the esophagus
5. Which of the following substances is most likely to cause gastritis?

A. Milk
B. Bicarbonate of soda, or baking soda
C. Enteric coated aspirin
D. Nonsteriodal anti-imflammatory drugs
6. Which of the following definitions best describes diverticulosis?

A. An inflamed outpouching of the intestine


B. A noninflamed outpouching of the intestine
C. The partial impairment of the forward flow of intestinal contents
D. An abnormal protrusion of an organ through the structure that usually holds it.
7. Which of the following types of diets is implicated in the development of diverticulosis?

A. Low-fiber diet
B. High-fiber diet
C. High-protein diet
D. Low-carbohydrate diet
8. Which of the following mechanisms can facilitate the development of diverticulosis into
diverticulitis?

A. Treating constipation with chronic laxative use, leading to dependence on laxatives


B. Chronic constipation causing an obstruction, reducing forward flow of intestinal
contents
C. Herniation of the intestinal mucosa, rupturing the wall of the intestine
D. Undigested food blocking the diverticulum, predisposing the area to bacteria invasion.
9. Which of the following symptoms indicated diverticulosis?

A. No symptoms exist
B. Change in bowel habits
C. Anorexia with low-grade fever
D. Episodic, dull, or steady midabdominal pain
10. Which of the following tests should be administered to a client suspected of having
diverticulosis?

A. Abdominal ultrasound
B. Barium enema
C. Barium swallow
D. Gastroscopy
11. Medical management of the client with diverticulitis should include which of the following
treatments?

A. Reduced fluid intake


B. Increased fiber in diet
C. Administration of antibiotics
D. Exercises to increase intra-abdominal pressure
12. Crohn’s disease can be described as a chronic relapsing disease. Which of the following areas
in the GI system may be involved with this disease?

A. The entire length of the large colon


B. Only the sigmoid area
C. The entire large colon through the layers of mucosa and submucosa
D. The small intestine and colon; affecting the entire thickness of the bowel
13. Which area of the alimentary canal is the most common location for Crohn’s disease?

A. Ascending colon
B. Descending colon
C. Sigmoid colon
D. Terminal ileum
14. Which of the following factors is believed to be linked to Crohn’s disease?

A. Constipation
B. Diet
C. Hereditary
D. Lack of exercise
15. Which of the following factors is believed to cause ulcerative colitis?

A. Acidic diet
B. Altered immunity
C. Chronic constipation
D. Emotional stress
16. Fistulas are most common with which of the following bowel disorders?

A. Crohn’s disease
B. Diverticulitis
C. Diverticulosis
D. Ulcerative colitis
17. Which of the following areas is the most common site of fistulas in client’s with Crohn’s
disease?

A. Anorectal
B. Ileum
C. Rectovaginal
D. Transverse colon
18. Which of the following associated disorders may a client with ulcerative colitis exhibit?

A. Gallstones
B. Hydronephrosis
C. Nephrolithiasis
D. Toxic megacolon
19. Which of the following associated disorders may the client with Crohn’s disease exhibit?

A. Ankylosing spondylitis
B. Colon cancer
C. Malabsorption
D. Lactase deficiency
20. Which of the following symptoms may be exhibited by a client with Crohn’s disease?

A. Bloody diarrhea
B. Narrow stools
C. N/V
D. Steatorrhea
21. Which of the following symptoms is associated with ulcerative colitis?

A. Dumping syndrome
B. Rectal bleeding
C. Soft stools
D. Fistulas
22. If a client had irritable bowel syndrome, which of the following diagnostic tests would
determine if the diagnosis is Crohn’s disease or ulcerative colitis?

A. Abdominal computed tomography (CT) scan


B. Abdominal x-ray
C. Barium swallow
D. Colonoscopy with biopsy
23. Which of the following interventions should be included in the medical management of
Crohn’s disease?

A. Increasing oral intake of fiber


B. Administering laxatives
C. Using long-term steroid therapy
D. Increasing physical activity
24. In a client with Crohn’s disease, which of the following symptoms should not be a direct result
from antibiotic therapy?

A. Decrease in bleeding
B. Decrease in temperature
C. Decrease in body weight
D. Decrease in the number of stools
25. Surgical management of ulcerative colitis may be performed to treat which of the following
complications?

A. Gastritis
B. Bowel herniation
C. Bowel outpouching
D. Bowel perforation
26. Which of the following medications is most effective for treating the pain associated with
irritable bowel disease?

A. Acetaminophen
B. Opiates
C. Steroids
D. Stool softeners
27. During the first few days of recovery from ostomy surgery for ulcerative colitis, which of the
following aspects should be the first priority of client care?

A. Body image
B. Ostomy care
C. Sexual concerns
D. Skin care
28. Colon cancer is most closely associated with which of the following conditions?

A. Appendicitis
B. Hemorrhoids
C. Hiatal hernia
D. Ulcerative colitis
29. Which of the following diets is most commonly associated with colon cancer?

A. Low-fiber, high fat


B. Low-fat, high-fiber
C. Low-protein, high-carbohydrate
D. Low carbohydrate, high protein
30. Which of the following diagnostic tests should be performed annually over age 50 to screen for
colon cancer?

A. Abdominal CT scan
B. Abdominal x-ray
C. Colonoscopy
D. Fecal occult blood test
31. Radiation therapy is used to treat colon cancer before surgery for which of the following
reasons?

A. Reducing the size of the tumor


B. Eliminating the malignant cells
C. Curing the cancer
D. Helping the bowel heal after surgery
32. Which of the following symptoms is a client with colon cancer most likely to exhibit?

A. A change in appetite
B. A change in bowel habits
C. An increase in body weight
D. An increase in body temperature
33. A client has just had surgery for colon cancer. Which of the following disorders might the
client develop?

A. Peritonitis
B. Diverticulosis
C. Partial bowel obstruction
D. Complete bowel obstruction
34. A client with gastric cancer may exhibit which of the following symptoms?

A. Abdominal cramping
B. Constant hunger
C. Feeling of fullness
D. Weight gain
35. Which of the following diagnostic tests may be performed to determine if a client has gastric
cancer?

A. Barium enema
B. Colonoscopy
C. Gastroscopy
D. Serum chemistry levels
36. A client with gastric cancer can expect to have surgery for resection. Which of the following
should be the nursing management priority for the preoperative client with gastric cancer?

A. Discharge planning
B. Correction of nutritional deficits
C. Prevention of DVT
D. Instruction regarding radiation treatment
37. Care for the postoperative client after gastric resection should focus on which of the following
problems?

A. Body image
B. Nutritional needs
C. Skin care
D. Spiritual needs
38. Which of the following complications of gastric resection should the nurse teach the client to
watch for?

A. Constipation
B. Dumping syndrome
C. Gastric spasm
D. Intestinal spasms
39. A client with rectal cancer may exhibit which of the following symptoms?

A. Abdominal fullness
B. Gastric fullness
C. Rectal bleeding
D. Right upper quadrant pain
40. A client with which of the following conditions may be likely to develop rectal cancer?

A. Adenomatous polyps
B. Diverticulitis
C. Hemorrhoids
D. Peptic ulcer disease
41. Which of the following treatments is used for rectal cancer but not for colon cancer?

A. Chemotherapy
B. Colonoscopy
C. Radiation
D. Surgical resection
42. Which of the following conditions is most likely to directly cause peritonitis?

A. Cholelithiasis
B. Gastritis
C. Perforated ulcer
D. Incarcerated hernia
43. Which of the following symptoms would a client in the early stages of peritonitis exhibit?

A. Abdominal distention
B. Abdominal pain and rigidity
C. Hyperactive bowel sounds
D. Right upper quadrant pain
44. Which of the following laboratory results would be expected in a client with peritonitis?

A. Partial thromboplastin time above 100 seconds


B. Hemoglobin level below 10 mg/dL
C. Potassium level above 5.5 mEq/L
D. White blood cell count above 15,000
45. Which of the following therapies is not included in the medical management of a client with
peritonitis?

A. Broad-spectrum antibiotics
B. Electrolyte replacement
C. I.V. fluids
D. Regular diet
46. Which of the following aspects is the priority focus of nursing management for a client with
peritonitis?

A. Fluid and electrolyte balance


B. Gastric irrigation
C. Pain management
D. Psychosocial issues
47. A client with irritable bowel syndrome is being prepared for discharge. Which of the following
meal plans should the nurse give the client?

A. Low fiber, low-fat


B. High fiber, low-fat
C. Low fiber, high-fat
D. High-fiber, high-fat
48. A client presents to the emergency room, reporting that he has been vomiting every 30 to 40
minutes for the past 8 hours. Frequent vomiting puts him at risk for which of the following?

A. Metabolic acidosis with hyperkalemia


B. Metabolic acidosis with hypokalemia
C. Metabolic alkalosis with hyperkalemia
D. Metabolic alkalosis with hypokalemia
49. Five days after undergoing surgery, a client develops a small-bowel obstruction. A Miller-
Abbott tube is inserted for bowel decompression. Which nursing diagnosis takes priority?

A. Imbalanced nutrition: Less than body requirements


B. Acute pain
C. Deficient fluid volume
D. Excess fluid volume
50. When teaching an elderly client how to prevent constipation, which of the following
instructions should the nurse include?

A. “Drink 6 glasses of fluid each day.”


B. “Avoid grain products and nuts.”
C. “Add at least 4 grams of brain to your cereal each morning.”
D. “Be sure to get regular exercise.”
51. In a client with diarrhea, which outcome indicates that fluid resuscitation is successful?

A. The client passes formed stools at regular intervals


B. The client reports a decrease in stool frequency and liquidity
C. The client exhibits firm skin turgor
D. The client no longer experiences perianal burning.
52. When teaching a community group about measures to prevent colon cancer, which instruction
should the nurse include?

A. “Limit fat intake to 20% to 25% of your total daily calories.”


B. “Include 15 to 20 grams of fiber into your daily diet.”
C. “Get an annual rectal examination after age 35.”
D. “Undergo sigmoidoscopy annually after age 50.”
53. A 30-year old client experiences weight loss, abdominal distention, crampy abdominal pain,
and intermittent diarrhea after birth of her 2nd child. Diagnostic tests reveal gluten-induced
enteropathy. Which foods must she eliminate from her diet permanently?
A. Milk and dairy products
B. Protein-containing foods
C. Cereal grains (except rice and corn)
D. Carbohydrates
54. After a right hemicolectomy for treatment of colon cancer, a 57-year old client is reluctant to
turn while on bed rest. Which action by the nurse would be appropriate?

A. Asking a co-worker to help turn the client


B. Explaining to the client why turning is important.
C. Allowing the client to turn when he’s ready to do so
D. Telling the client that the physician’s order states he must turn every 2 hours
55. A client has a percutaneous endoscopic gastrostomy tube inserted for tube feedings. Before
starting a continuous feeding, the nurse should place the client in which position?

A. Semi-Fowlers
B. Supine
C. Reverse Trendelenburg
D. High Fowler’s
56. An enema is prescribed for a client with suspected appendicitis. Which of the following actions
should the nurse take?

A. Prepare 750 ml of irrigating solution warmed to 100*F


B. Question the physician about the order
C. Provide privacy and explain the procedure to the client
D. Assist the client to left lateral Sim’s position
57. The client being seen in a physician’s office has just been scheduled for a barium swallow the
next day. The nurse writes down which of the following instructions for the client to follow before
the test?

A. Fast for 8 hours before the test


B. Eat a regular supper and breakfast
C. Continue to take all oral medications as scheduled.
D. Monitor own bowel movement pattern for constipation
58. The nurse is monitoring a client for the early signs of dumping syndrome. Which symptom
indicates this occurrence?

A. Abdominal cramping and pain


B. Bradycardia and indigestion
C. Sweating and pallor
D. Double vision and chest pain
59. The nurse is preparing a discharge teaching plan for the client who had an umbilical hernia
repair. Which of the following would the nurse include in the plan?

A. Restricting pain medication


B. Maintaining bedrest
C. Avoiding coughing
D. Irrigating the drain
60. The nurse is caring for a hospitalized client with a diagnosis of ulcerative colitis. Which
finding, if noted on assessment of the client, would the nurse report to the physician?

A. Bloody diarrhea
B. Hypotension
C. A hemoglobin of 12 mg/dL
D. Rebound tenderness
61. The nurse is reviewing the record of a client with Crohn’s disease. Which of the following
stool characteristics would the nurse expect to note documented on the client’s record?

A. Chronic constipation
B. Diarrhea
C. Constipation alternating with diarrhea
D. Stool constantly oozing from the rectum
62. The nurse is performing a colostomy irrigation on a client. During the irrigation, a client begins
to complain of abdominal cramps. Which of the following is the most appropriate nursing action?

A. Notify the physician


B. Increase the height of the irrigation
C. Stop the irrigation temporarily.
D. Medicate with dilaudid and resume the irrigation
63. The nurse is teaching the client how to perform a colostomy irrigation. To enhance the
effectiveness of the irrigation and fecal returns, what measure should the nurse instruct the client to
do?

A. Increase fluid intake


B. Reduce the amount of irrigation solution
C. Perform the irrigation in the evening
D. Place heat on the abdomen
64. The nurse is reviewing the physician’s orders written for a client admitted with acute
pancreatitis. Which physician order would the nurse question if noted on the client’s chart?
A. NPO status
B. Insert a nasogastric tube
C. An anticholinergic medication
D. Morphine for pain
65. The nurse is doing an admission assessment on a client with a history of duodenal ulcer. To
determine whether the problem is currently active, the nurse would assess the client for which of
the following most frequent symptom(s) of duodenal ulcer?

A. Pain that is relieved by food intake


B. Pain that radiated down the right arm
C. N/V
D. Weight loss
66. The nurse instructs the ileostomy client to do which of the following as a part of essential care
of the stoma?

A. Cleanse the peristomal skin meticulously


B. Take in high-fiber foods such as nuts
C. Massage the area below the stoma
D. Limit fluid intake to prevent diarrhea.
67. The client who has undergone creation of a colostomy has a nursing diagnosis of Disturbed
body image. The nurse would evaluate that the client is making the most significant progress
toward identified goals if the client:

A. Watches the nurse empty the colostomy bag


B. Looks at the ostomy site
C. Reads the ostomy product literature
D. Practices cutting the ostomy appliance
68. The nurse is assessing for stoma prolapse in a client with a colostomy. The nurse would
observe which of the following if stoma prolapse occurred?

A. Sunken and hidden stoma


B. Dark- and bluish-colored stoma
C. Narrowed and flattened stoma
D. Protruding stoma
69. The client with a new colostomy is concerned about the odor from the stool in the ostomy
drainage bag. The nurse teaches the client to include which of the following foods in the diet to
reduce odor?

A. Yogurt
B. Broccoli
C. Cucumbers
D. Eggs
70. The nurse has given instructions to the client with an ileostomy about foods to eat to thicken
the stool. The nurse determines that the client needs further instructions if the client stated to eat
which of the following foods to make the stools less watery?
A. Pasta
B. Boiled rice
C. Bran
D. Low-fat cheese
71. The client has just had surgery to create an ileostomy. The nurse assesses the client in the
immediate post-op period for which of the following most frequent complications of this type of
surgery?

A. Intestinal obstruction
B. Fluid and electrolyte imbalance
C. Malabsorption of fat
D. Folate deficiency
72. The nurse is doing pre-op teaching with the client who is about to undergo creation of a Kock
pouch. The nurse interprets that the client has the best understanding of the nature of the surgery if
the client makes which of the following statements?

A. “I will need to drain the pouch regularly with a catheter.”


B. “I will need to wear a drainage bag for the rest of my life.”
C. “The drainage from this type of ostomy will be formed.”
D. “I will be able to pass stool from my rectum eventually.”
73. The client with a colostomy has an order for irrigation of the colostomy. The nurse used which
solution for irrigation?

A. Distilled water
B. Tap water
C. Sterile water
D. Lactated Ringer’s
74. A nurse is monitoring a client admitted to the hospital with a diagnosis of appendicitis. The
client is scheduled for surgery in 2 hours. The client begins to complain of increased abdominal
pain and begins to vomit. On assessment the nurse notes that the abdomen is distended and the
bowel sounds are diminished. Which of the following is the most appropriate nursing intervention?

A. Administer dilaudid
B. Notify the physician
C. Call and ask the operating room team to perform the surgery as soon as possible
D. Reposition the client and apply a heating pad on a warm setting to the client’s abdomen.
75. The client has been admitted with a diagnosis of acute pancreatitis. The nurse would assess this
client for pain that is:

A. Severe and unrelenting, located in the epigastric area and radiating to the back.
B. Severe and unrelenting, located in the left lower quadrant and radiating to the groin.
C. Burning and aching, located in the epigastric area and radiating to the umbilicus.
D. Burning and aching, located in the left lower quadrant and radiating to the hip.
76. The client with Crohn’s disease has a nursing diagnosis of acute pain. The nurse would teach
the client to avoid which of the following in managing this problem?
A. Lying supine with the legs straight
B. Massaging the abdomen
C. Using antispasmodic medication
D. Using relaxation techniques
77. A client with ulcerative colitis has an order to begin salicylate medication to reduce
inflammation. The nurse instructs the client to take the medication:

A. 30 minutes before meals


B. On an empty stomach
C. After meals
D. On arising
78. During the assessment of a client’s mouth, the nurse notes the absence of saliva. The client is
also complaining of pain near the area of the ear. The client has been NPO for several days because
of the insertion of a NG tube. Based on these findings, the nurse suspects that the client is
developing which of the following mouth conditions?

A. Stomatitis
B. Oral candidiasis
C. Parotitis
D. Gingivitis
79. The nurse evaluates the client’s stoma during the initial post-op period. Which of the following
observations should be reported immediately to the physician?

A. The stoma is slightly edematous


B. The stoma is dark red to purple
C. The stoma oozes a small amount of blood
D. The stoma does not expel stool
80. When planning care for a client with ulcerative colitis who is experiencing symptoms, which
client care activities can the nurse appropriately delegate to a unlicensed assistant? Select all that
apply.

A. Assessing the client’s bowel sounds


B. Providing skin care following bowel movements
C. Evaluating the client’s response to antidiarrheal medications
D. Maintaining intake and output records
E. Obtaining the client’s weight.
81. Which goal of the client’s care should take priority during the first days of hospitalization for
an exacerbation of ulcerative colitis?

A. Promoting self-care and independence


B. Managing diarrhea
C. Maintaining adequate nutrition
D. Promoting rest and comfort
82. A client’s ulcerative colitis symptoms have been present for longer than 1 week. The nurse
recognizes that the client should be assessed carefully for signs of which of the following
complications?
A. Heart failure
B. DVT
C. Hypokalemia
D. Hypocalcemia
83. A client who has ulcerative colitis has persistent diarrhea. He is thin and has lost 12 pounds
since the exacerbation of his ulcerative colitis. The nurse should anticipate that the physician will
order which of the following treatment approaches to help the client meet his nutritional needs?

A. Initiate continuous enteral feedings


B. Encourage a high protein, high-calorie diet
C. Implement total parenteral nutrition
D. Provide six small meals a day.
84. Digoxin preparations and absorbents should not be given simultaneously. As a nurse, you are
aware that if these agents are given simultaneously, which of the following will occur?

A. Increased absorption of digoxin


B. Decreased absorption of digoxin
C. Increased absorption of the absorbent
D. Decreased absorption of the absorbent
85. When used with hyperacidic disorders of the stomach, antacids are given to elevate the gastric
pH to:

A. 2.0
B. 4.0
C. 6.0
D. >8.0
86. One of your patients is receiving digitalis orally and is also to receive an antacid at the same
time. Your most appropriate action, based on the pharmacokinetics of antacids, is to:

A. Delay the digitalis for 1 to 2 hours until the antacid is absorbed


B. Give the antacid at least 2 to 4 hours before administering the digitalis
C. Administer both medications as ordered and document in nurse’s notes
D. Contact the physician regarding the drug interaction and request a change in the time of
dosing of the drugs.
87. The nurse would teach patients that antacids are effective in treatment of hyperacidity because
they:

A. Neutralize gastric acid


B. Decrease stomach motility
C. Decrease gastric pH
D. Decrease duodenal pH
88. The nurse would monitor for which of the following adverse reactions to aluminum-containing
antacids such as aluminum hydroxide (Amphojel)?

A. Diarrhea
B. Constipation
C. GI upset
D. Fluid retention
89. The nurse would question an order for which type of antacid in patients with chronic renal
failure?

A. Aluminum-containing antacids
B. Calcium-containing antacids
C. Magnesium-containing antacids
D. All of the above.
90. The nurse would monitor a patient using sodium bicarbonate to treat gastric hyperacidity for
signs and symptoms of:

A. Metabolic alkalosis
B. Metabolic acidosis
C. Hyperkalemia
D. Hypercalcemia
91. Which of the following nursing diagnoses is appropriate for a patient receiving famotidine
(Pepcid)?

A. Increased risk for infection due to immunosuppresion


B. Potential risk for bleeding related to thrombocytopenia.
C. Aleration in urinary elimination related to retention
D. Alteration in tissue perfusion related to hypertension
92. Histamine2-receptor antagonists:
A. Compete with histamine for binding sites on the parietal cells
B. Irreversibly bind to H+/K+ATPase
C. Cause a decrease in stomach pH
D. Decrease signs and symptoms of allergies related to histamine release
93. Proton pump inhibitors:

A. Gastric ulcer formation


B. GERD
C. Achlorhydria
D. Diverticulosis
94. A patient unable to tolerate oral medications may be prescribed which of the following proton
pump inhibitors to be administered intravenously?

A. lansoprazole (Prevacid)
B. omeprazole (Prilosec)
C. pantoprazole (Protonix)
D. esomeprazole (Nexium)
95. When administering sucralfate (Carafate) to a patient with a nasogastric tube, it is important to:

A. Crush the tablet into a fine powder before mixing with water
B. Administer with a bolus tube feeding
C. Allow the tablet to dissolve in water before administering
D. Administer with an antacid for maximum benefit
96. Sucralfate (Carafate) achieves a therapeutic effect by:

A. Neutralizing gastric acid


B. Enhancing gastric absorption
C. Forming a protective barrier around gastric mucosa
D. Inhibiting gastric acid secretion
97. To avoid fecal impaction, psyllium (Metamucil) should be administered with at least how
many ounces of fluid?

A. 4
B. 6
C. 8
D. 10
98. Bismuth subsalicylate (Pepto-Bismol), as an absorbent, has which of the following
mechanisms of action?

A. Decreased GI motility
B. Decreased gastric secretions
C. Increased fluid absorption
D. Binding to diarrhea-causing bacteria for excretion
99. Side effects of loperamide (Imodium) include all of the following except?

A. Diarrhea
B. epigastric pain
C. Dry mouth
D. Anorexia
100. The mechanism of action of diphenoxylate (Lotomil) is:

A. An increase in intestinal excretion of water


B. An increase in intestinal motility
C. A decrease in peristalsis in the intestinal wall
D. A decrease in the reabsorption of water in the bowel
101. Milk of Magnesium alleviates constipation by:

A. Increasing water absorption into the colon


B. Lubricating the passage of stool
C. Increasing bulk within the colon
D. Stimulating nerves that regulate defecation
102. Lactulose, a hyperosmotic laxative, also functions to:

A. Decrease peripheral edema


B. Correct sodium imbalances
C. Reduce ammonia levels
D. Alleviate galactose intolerance
Answers and Rationales
1. A. A fecalith is a fecal calculus, or stone, that occludes the lumen of the appendix and is
the most common cause of appendicitis. Bowel wall swelling, kinking of the appendix,
and external occlusion, not internal occlusion, of the bowel by adhesions can also be
causes of appendicitis.
2. D. The pain begins in the epigastrium or periumbilical region, then shifts to the right
lower quadrant and becomes steady. The pain may be moderate to severe.
3. D. The focus of care is to assess for peritonitis, or inflammation of the peritoneal cavity.
Peritonitis is most commonly caused by appendix rupture and invasion of bacteria,
which could be lethal. The client with appendicitis will have pain that should be
controlled with analgesia. The nurse should discourage oral intake in preparation of
surgery. Discharge teaching is important; however, in the acute phase, management
should focus on minimizing preoperative complications and recognizing when such may
be occurring.
4. C. Gastritis is an inflammation of the gastric mucosa that may be acute (often resulting
from exposure to local irritants) or chronic (associated with autoimmune infections or
atrophic disorders of the stomach). Erosion of the mucosa results in ulceration.
Inflammation of a diverticulum is called diverticulitis; reflux of stomach acid is known
as gastroesophageal disease.
5. D. NSAIDS are a common cause of gastritis because they inhibit prostaglandin
synthesis. Milk, once thought to help gastritis, has little effect on the stomach mucosa.
Bicarbonate of soda, or baking soda, may be used to neutralize stomach acid, but it
should be used cautiously because it may lead to metabolic acidosis. ASA with enteric
coating shouldn’t contribute significantly to gastritis because the coating limits the
aspirin’s effect on the gastric mucosa.
6. B. Diverticulosis involves a noninflamed outpouching of the intestine. Diverticulitis
involves an inflamed outpouching. The partial impairment of forward flow of the
intestine is an obstruction; abnormal protrusion of an organ is a hernia.
7. A. Low-fiber diets have been implicated in the development of diverticula because these
diets decrease the bulk in the stool and predispose the person to the development of
constipation. A high-fiber diet is recommended to help prevent diverticulosis. A high-
protein or low-carbohydrate diet has no effect on the development of diverticulosis.
8. D. Undigested food can block the diverticulum, decreasing blood supply to the area and
predisposing the area to invasion of bacteria. Chronic laxative use is a common problem
in elderly clients, but it doesn’t cause diverticulitis. Chronic constipation can cause an
obstruction—not diverticulitis. Herniation of the intestinal mucosa causes an intestinal
perforation.
9. A. Diverticulosis is an asymptomatic condition. The other choices are signs and
symptoms of diverticulitis.
10.B. A barium enema will cause diverticula to fill with barium and be easily seen on x-ray.
An abdominal US can tell more about structures, such as the gallbladder, liver, and
spleen, than the intestine. A barium swallow and gastroscopy view upper GI structures.
11.C. Antibiotics are used to reduce the inflammation. The client isn’t typically isn’t
allowed anything orally until the acute episode subsides. Parenteral fluids are given until
the client feels better; then it’s recommended that the client drink eight 8-ounce glasses
of water per day and gradually increase fiber in the diet to improve intestinal motility.
During the acute phase, activities that increase intra-abdominal pressure should be
avoided to decrease pain and the chance of intestinal obstruction.
12.D. Crohn’s disease can involve any segment of the small intestine, the colon, or both,
affecting the entire thickness of the bowel. Answers A and C describe ulcerative colitis,
answer B is too specific and therefore, not likely.
13.D. Studies have shown that the terminal ileum is the most common site for recurrence in
clients with Crohn’s disease. The other areas may be involved but aren’t as common.
14.C. Although the definite cause of Crohn’s disease is unknown, it’s thought to be
associated with infectious, immune, or psychological factors. Because it has a higher
incidence in siblings, it may have a genetic cause.
15.B. Several theories exist regarding the cause of ulcerative colitis. One suggests altered
immunity as the cause based on the extraintestinal characteristics of the disease, such as
peripheral arthritis and cholangitis. Diet and constipation have no effect on the
development of ulcerative colitis. Emotional stress can exacerbate the attacks but isn’t
believed to be the primary cause.
16.A. The lesions of Crohn’s disease are transmural; that is, they involve all thickness of
the bowel. These lesions may perforate the bowel wall, forming fistulas with adjacent
structures. Fistulas don’t develop in diverticulitis or diverticulosis. The ulcers that occur
in the submucosal and mucosal layers of the intestine in ulcerative colitis usually don’t
progress to fistula formation as in Crohn’s disease.
17.A. Fistulas occur in all these areas, but the anorectal area is most common because of the
relative thinness of the intestinal wall in this area.
18.D. Toxic megacolon is extreme dilation of a segment of the diseased colon caused by
paralysis of the colon, resulting in complete obstruction. This disorder is associated with
both Crohn’s disease and ulcerative colitis. The other disorders are more commonly
associated with Crohn’s disease.
19.C. Because of the transmural nature of Crohn’s disease lesions, malaborption may occur
with Crohn’s disease. Ankylosing spondylitis and colon cancer are more commonly
associated with ulcerative colitis. Lactase deficiency is caused by a congenital defect in
which an enzyme isn’t present.
20.D. Steatorrhea from malaborption can occur with Crohn’s disease. N/V, and bloody
diarrhea are symptoms of ulcerative colitis. Narrow stools are associated with
diverticular disease.
21.B. In ulcerative colitis, rectal bleeding is the predominant symptom. Soft stools are more
commonly associated with Crohn’s disease, in which malabsorption is more of a
problem. Dumping syndrome occurs after gastric surgeries. Fistulas are associated with
Crohn’s disease.
22.D. A colonoscopy with biopsy can be performed to determine the state of the colon’s
mucosal layers, presence of ulcerations, and level of cytologic development. An
abdominal x-ray or CT scan wouldn’t provide the cytologic information necessary to
diagnose which disease it is. A barium swallow doesn’t involve the intestine.
23.C. Management of Crohn’s disease may include long-term steroid therapy to reduce the
inflammation associated with the deeper layers of the bowel wall. Other management
focuses on bowel rest (not increasing oral intake) and reducing diarrhea with
medications (not giving laxatives). The pain associated with Crohn’s disease may
require bed rest, not an increase in physical activity.
24.C. A decrease in body weight may occur during therapy due to inadequate dietary
intake, but isn’t related to antibiotic therapy. Effective antibiotic therapy will be noted
by a decrease in temperature, number of stools, and bleeding.
25.D. Perforation, obstruction, hemorrhage, and toxic megacolon are common
complications of ulcerative colitis that may require surgery. Herniation and gastritis
aren’t associated with irritable bowel diseases, and outpouching of the bowel is
diverticulosis.
26.C. The pain with irritable bowel disease is caused by inflammation, which steroids can
reduce. Stool softeners aren’t necessary. Acetaminophen has little effect on the pain, and
opiate narcotics won’t treat its underlying cause (I feel this is untrue—dilaudid will
helpanything!)
27.B. Although all of these are concerns the nurse should address, being able to safely
manage the ostomy is crucial for the client before discharge.
28.D. Chronic ulcerative colitis, granulomas, and familial polposis seem to increase a
person’s chance of developing colon cancer. The other conditions listed have no known
effect on colon cancer risk.
29.A. A low-fiber, high-fat diet reduced motility and increases the chance of constipation.
The metabolic end products of this type of diet are carcinogenic. A low-fat, high-fiber
diet is recommended to prevent colon cancer.
30.D. Surface blood vessels of polyps and cancers are fragile and often bleed with the
passage of stools. Abdominal x-ray and CT scan can help establish tumor size and
metastasis. A colonoscopy can help locate a tumor as well as polyps, which can be
removed before they become malignant.
31.A. Radiation therapy is used to treat colon cancer before surgery to reduce the size of the
tumor, making it easier to be resected. Radiation therapy isn’t curative, can’t eliminate
the malignant cells (though it helps define tumor margins), can could slow postoperative
healing.
32.B. The most common complaint of the client with colon cancer is a change in bowel
habits. The client may have anorexia, secondary abdominal distention, or weight loss.
Fever isn’t associated with colon cancer.
33.A. Bowel spillage could occur during surgery, resulting in peritonitis. Complete or
partial bowel obstruction may occur before bowel resection. Diverticulosis doesn’t result
from surgery or colon cancer.
34.C. The client with gastric cancer may report a feeling of fullness in the stomach, but not
enough to cause him to seek medical attention. Abdominal cramping isn’t associated
with gastric cancer. Anorexia and weight loss (not increased hunger or weight gain) are
common symptoms of gastric cancer.
35.A. gastroscopy will allow direct visualization of the tumor. A colonoscopy or a barium
enema would help diagnose colon cancer. Serum chemistry levels don’t contribute data
useful to the assessment of gastric cancer.
36.B. Client’s with gastric cancer commonly have nutritional deficits and may be cachectic.
Discharge planning before surgery is important, but correcting the nutrition deficit is a
higher priority. At present, radiation therapy hasn’t been proven effective for gastric
cancer, and teaching about it preoperatively wouldn’t be appropriate. Prevention of DVT
also isn’t a high priority to surgery, though it assumes greater importance after surgery.
37.B. After gastric resection, a client may require total parenteral nutrition or jejunostomy
tube feedings to maintain adequate nutritional status.
38.B. Dumping syndrome is a problem that occurs postprandially after gastric resection
because ingested food rapidly enters the jejunum without proper mixing and without the
normal duodenal digestive processing. Diarrhea, not constipation, may also be a
symptom. Gastric or intestinal spasms don’t occur, but antispasmidics may be given to
slow gastric emptying.
39.C. Rectal bleeding is a common symptom of rectal cancer. Rectal cancer may be missed
because other conditions such as hemorrhoids can cause rectal bleeding. Abdominal
fullness may occur with colon cancer, gastric fullness may occur with gastric cancer, and
right upper quadrant pain may occur with liver cancer.
40.A. A client with adenomatous polyps has a higher risk for developing rectal cancer than
others do. Clients with diverticulitis are more likely to develop colon cancer.
Hemorrhoids don’t increase the chance of any type of cancer. Clients with peptic ulcer
disease have a higher incidence of gastric cancer.
41.C. A client with rectal cancer can expect to have radiation therapy in addition to
chemotherapy and surgical resection of the tumor. A colonoscopy is performed to
diagnose the disease. Radiation therapy isn’t usually indicated in colon cancer.
42.C. The most common cause of peritonitis is a perforated ulcer, which can pour
contaminates into the peritoneal cavity, causing inflammation and infection within the
cavity. The other conditions don’t by themselves cause peritonitis. However, if
cholelithiasis leads to rupture of the gallbladder, gastritis leads to erosion of the stomach
wall, or an incarcerated hernia leads to rupture of the intestines, peritonitis may develop.
43.B. Abdominal pain causing rigidity of the abdominal muscles is characteristic of
peritonitis. Abdominal distention may occur as a late sign but not early on. Bowel
sounds may be normal or decreased but not increased. Right upper quadrant pain is
chatacteristic of cholecystitis or hepatitis.
44.D. Because of infection, the client’s WBC count will be elevated. A hemoglobin level
below 10 mg/dl may occur from hemorrhage. A PT time longer than 100 seconds may
suggest disseminated intravascular coagulation, a serious complication of septic shock.
A potassium level above 5.5 mEq/L may indicate renal failure.
45.D. The client with peritonitis usually isn’t allowed anything orally until the source of
peritonitis is confirmed and treated. The client also requires broad-spectrum antibiotics
to combat the infection. I.V. fluids are given to maintain hydration and hemodynamic
stability and to replace electrolytes.
46.A. Peritonitis can advance to shock and circulatory failure, so fluid and electrolyte
balance is the priority focus of nursing management. Gastric irrigation may be needed
periodically to ensure patency of the nasogastric tube. Although pain management is
important for comfort and psychosocial care will address concerns such as anxiety,
focusing on fluid and electrolyte imbalance will maintain hemodynamic stability.
47.B. The client with irritable bowel syndrome needs to be on a diet that contains at least 25
grams of fiber per day. Fatty foods are to be avoided because they may precipitate
symptoms.
48.D. Gastric acid contains large amounts of potassium, chloride, and hydrogen ions.
Excessive loss of these substances, such as from vomiting, can lead to metabolic
alkalosis and hypokalemia.
49.C. Fluid shifts to the site of the bowel obstruction, causing a fluid deficit in the
intravascular spaces. If the obstruction isn’t resolved immediately, the client may
experience an imbalanced nutritional status (less than body requirements); however,
deficient fluid volume takes priority. The client may also experience pain, but that
nursing diagnosis is also of lower priority than deficient fluid volume.
50.D. Exercise helps prevent constipation. Fluids and dietary fiber promote normal bowel
function. The client should drink eight to ten glasses of fluid each day. Although adding
bran to cereal helps prevent constipation by increasing dietary fiber, the client should
start with a small amount and gradually increase the amount as tolerated to a maximum
of 2 grams a day.
51.C. A client with diarrhea has a nursing diagnosis of Deficient fluid volume related to
excessive fluid loss in the stool. Expected outcomes include firm skin turgor, moist
mucous membranes, and urine output of at least 30 ml/hr. The client also has a nursing
diagnosis of diarrhea, with expected outcomes of passage of formed stools at regular
intervals and a decrease in stool frequency and liquidity. The client is at risk for
impaired skin integrity related to irritation from diarrhea; expected outcomes for this
diagnosis include absence of erythema in perianal skin and mucous membranes and
absence of perianal tenderness or burning.
52.A. To help prevent colon cancer, fats should account for no more than 20% to 25% of
total daily calories and the diet should include 25 to 30 grams of fiber per day. A digital
rectal examination isn’t recommended as a stand-alone test for colorectal cancer. For
colorectal cancer screening, the American Cancer society advises clients over age 50 to
have a flexible sigmoidoscopy every 5 years, yearly fecal occult blood tests, yearly fecal
occult blood tests PLUS a flexible sigmoidoscopy every 5 years, a double-contrast
barium enema every 5 years, or a colonoscopy every 10 years.
53.C. To manage gluten-induced enteropathy, the client must eliminate gluten, which
means avoiding all cereal grains except for rice and corn. In initial disease management,
clients eat a high calorie, high-protein diet with mineral and vitamin supplements to help
normalize nutritional status.
54.B. The appropriate action is to explain the importance of turning to avoid postoperative
complications. Asking a coworker to help turn the client would infringe on his rights.
Allowing him to turn when he’s ready would increase his risk for postoperative
complications. Telling him he must turn because of the physician’s orders would put
him on the defensive and exclude him from participating in care decision.
55.A. To prevent aspiration of stomach contents, the nurse should place the client in semi-
Fowler’s position. High Fowler’s position isn’t necessary and may not be tolerated as
well as semi-Fowler’s.
56.B. Enemas are contraindicated in an acute abdominal condition of unknown origin as
well as after recent colon or rectal surgery or myocardial infarction. The other answers
are correct only when enema administration is appropriate.
57.A. A barium swallow is an x-ray study that uses a substance called barium for contrast to
highlight abnormalities in the GI tract. The client should fast for 8 to 12 hours before the
test, depending on the physician instructions. Most oral medications also are withheld
before the test. After the procedure the nurse must monitor for constipation, which can
occur as a result of the presence of barium in the GI tract.
58.C. Early manifestations of dumping syndrome occur 5 to 30 minutes after eating.
Symptoms include vertigo, tachycardia, syncope, sweating, pallor, palpitations, and the
desire to lie down.
59.C. Bedrest is not required following this surgical procedure. The client should take
analgesics as needed and as prescribed to control pain. A drain is not used in this
surgical procedure, although the client may be instructed in simple dressing changes.
Coughing is avoided to prevent disruption of the tissue integrity, which can occur
because of the location of this surgical procedure.
60.D. Rebound tenderness may indicate peritonitis. Blood diarrhea is expected to occur in
ulcerative colitis. Because of the blood loss, the client may be hypotensive and the
hemoglobin level may be lower than normal. Signs of peritonitis must be reported to the
physician.
61.B. Crohn’s disease is characterized by nonbloody diarrhea of usually not more than four
to five stools daily. Over time, the diarrhea episodes increase in frequency, duration and
severity. The other option are not associated with diarrhea.
62.C. If cramping occurs during a colostomy irrigation, the irrigation flow is stopped
temporarily and the client is allowed to rest. Cramping may occur from an infusion that
is too rapid or is causing too much pressure. Increasing the height of the irrigation will
cause further discomfort. The physician does not need to be notified. Medicating the
client for pain is not the most appropriate action (damn).
63.A. To enhance effectiveness of the irrigation and fecal returns, the client is instructed to
increase fluid intake and prevent constipation.
64.D. Meperidine (Demerol) rather than morphine is the medication of choice because
morphine can cause spasm in the sphincter of Oddi.
65.A. The most frequent symptom of duodenal ulcer is pain that is relieved by food intake.
These clients generally describe the pain as burning, heavy, sharp, or “hungry” pain that
often localizes in the midepigastric area. The client with duodenal ulcer usually does not
experience weight loss or N/V. These symptoms are usually more typical in the client
with a gastric ulcer.
66.A. The peristomal skin must receive meticulous cleansing because the ileostomy
drainage has more enzymes and is more caustic to the skin than colostomy drainage.
Foods such as nuts and those with seeds will pass through the ileostomy. The client
should be taught that these foods will remain undigested. The area below the ileostomy
may be massaged if needed if the ileostomy becomes blocked by high fiber foods. Fluid
intake should be maintained to at least six to eight glasses of water per day to prevent
dehydration.
67.D. The client is expected to have a body image disturbance after colostomy. The client
progresses through normal grieving stages to adjust to this change. The client
demonstrates the greatest deal of acceptance when the client participates in the actual
colostomy care. Each of the incorrect options represents an interest in colostomy care
but is a passive activity. The correct option shows the client is participating in self-care.
68.D. A prolapsed stoma is one which the bowel protruded through the stoma. A stoma
retraction is characterized by sinking of the stoma. Ischemia of the stoma would be
associated with dusky or bluish color. A stoma with a narrowed opening at the level of
the skin or fascia is said to be stenosed.
69.A. The client should be taught to include deodorizing foods in the diet, such a beet
greens, parsley, buttermilk, and yogurt. Spinach also reduces odor but is a gas forming
food as well. Broccoli, cucumbers, and eggs are gas forming foods.
70.C. Foods that help thicken the stool of the client with an ileostomy include pasta, boiled
rice, and low-fat cheese. Bran is high in dietary fiber and thus will increase output of
watery stool by increasing propulsion through the bowel. Ileostomy output is liquid.
Addition or elimination of various foods can help thicken or loosen this liquid drainage.
71.B. A major complication that occurs most frequent following an ileostomy is fluid and
electrolyte imbalance. The client requires constant monitoring of intake and output to
prevent this from happening. Losses require replacement by intravenous infusion until
the client can tolerate a diet orally. Intestinal obstruction is a less frequent complication.
Fat malabsorption and folate deficiency are complications that could occur later in the
postoperative period.
72.A. A Kock pouch is a continent ileostomy. As the ileostomy begins to function, the
client drains it every 3 to 4 hours and then decreases the draining to about 3 times a day
or as needed when full. The client does not need to wear a drainage bag but should wear
an absorbent dressing to absorb mucus drainage from the stoma. Ileostomy drainage is
liquid. The client would be able to pass stool only from the rectum if an ileal-anal pouch
or anastamosis were created. This type of operation is a two-stage procedure.
73.B. Warm tap water or saline solution is used to irrigate a colostomy. If the tap water is
not suitable for drinking, then bottled water should be used.
74.B. Based on the signs and symptoms presented in the question, the nurse should suspect
peritonitis and should notify the physician. Administering pain medication is not an
appropriate intervention. Heat should never be applied to the abdomen of a client with
suspected appendicitis. Scheduling surgical time is not within the scope of nursing
practice, although the physician probably would perform the surgery earlier than the
prescheduled time.
75.A. The pain associated with acute pancreatitis is often severe and unrelenting, is located
in the epigastric region, and radiates to the back.
76.A. The pain associated with Crohn’s disease is alleviated by the use of analgesics and
antispasmodics and also is reduced by having the client practice relaxation techniques,
applying local cold or heat to the abdomen, massaging the abdomen, and lying with the
legs flexed. Lying with the legs extended is not useful because it increases the muscle
tension in the abdomen, which could aggravate the inflamed intestinal tissues as the
abdominal muscles are stretched.
77.C. Salicylate compounds act by inhibiting prostaglandin synthesis and reducing
inflammation. The nurse teaches the client to take the medication with a full glass of
water and to increase fluid intake throughout the day. This medication needs to be taken
after meals to reduce GI irritation.
78.D. The lack of saliva, pain near the area of the ear, and the prolonged NPO status of the
client should lead the nurse to suspect the development of parotitis, or inflammation of
the parotid gland. Parotitis usually develops in cases of dehydration combined with poor
oral hygiene or when clients have been NPO for an extended period. Preventative
measures include the use of sugarless hard candy or gum to stimulate saliva production,
adequate hydration, and frequent mouth care. Stomatitis (inflammation of the mouth)
produces excessive salivation and a sore mouth.
79.B. A dark red to purple stoma indicates inadequate blood supply. Mild edema and slight
oozing of blood are normal in the early post-op period. The colostomy would typically
not begin functioning until 2-4 days after surgery.
80.B, D, and E. The nurse can delegate the following basic care activities to the unlicensed
assistant: providing skin care following bowel movements, maintaining intake and
output records, and obtaining the client’s weight. Assessing the client’s bowel sounds
and evaluating the client’s response to medication are registered nurse activities that
cannot be delegated.
81.B. Diarrhea is the primary symptom in an exacerbation of ulcerative colitis, and
decreasing the frequency of stools is the first goal of treatment. The other goals are
ongoing and will be best achieved by halting the exacerbation. The client may receive
antidiarrheal medications, antispasmodic agents, bulk hydrophilic agents, or anti-
inflammatory drugs.
82.C. Excessive diarrhea causes significant depletion of the body’s stores of sodium and
potassium as well as fluid. The client should be closely monitored for hypokalemia and
hyponatremia. Ulcerative colitis does not place the client at risk for heart failure, DVT,
or hypocalcemia.
83.C. Food will be withheld from the client with severe symptoms of ulcerative colitis to
rest the bowel. To maintain the client’s nutritional status, the client will be started on
TPN. Enteral feedings or dividing the diet into 6 small meals does not allow the bowel to
rest. A high-calorie, high-protein diet will worsen the client’s symptoms.
84.B.
85.A.
86.D.
87.A. Antacids work by neutralizing gastric acid, which would cause an increase in pH.
They do not affect gastric motility.
88.B. Aluminum- and calcium-containing antacids cause constipation, magnesium-
containing antacids cause diarrhea, and sodium-containing antacids cause sodium and
fluid retention.
89.C. Magnesium-containing antacids can cause hypermagnesemia in patients with chronic
renal failure. Aluminum-containing antacids may be used as a phosphate binder in
patients with chronic renal failure. Calcium-containing antacids are also appropriate
because these patients may be hypocalcemic.
90.A. Solutions containing sodium bicarbonate (a base) can cause metabolic alkalosis.
Serum K and serum calcium would decrease with alkalosis, not increase.
91.B. A serious side effect of famotidine is thrombocytopenia, which is manifested by a
decrease in platelet count and an increased risk of bleeding.
92.A. Histamine receptor blocking agents decrease gastric acid by competing with
histamine for binding sites on the parietal cells.
93.C. Because the proton pump inhibitors stop the final step of acid secretion, they can
block up to 90% of acid secretion, leading to achlorhydria (without acid).
94.C. Pantoprazole is the only proton pump inhibitor that is available for intravenous
administration. The other medications in this category may only be administered orally.
95.C. It is important to give sucralfate on an empty stomach so that it may dissolve and
form a protective barrier over the gastric mucosa. The tablet form will not dissolve in
water when crushed; it must be left whole and allowed to dissolve. Crushing the
medication so that it will not dissolve could lead to clogging of the nasogastric tube and
decreased effectiveness of the drug.
96.C. Sucralfate has a local effect only on the gastric mucosa. It forms a pastelike substance
in the stomach, which adheres to the gastric lining, protecting against adverse effects
related to gastric acid. It also stimulates healing of any ulcerated areas of the gastric
mucosa.
97.C. Bulk-forming laxatives must be given with at least 8 ounces of liquid plus additional
liquid each day to prevent intestinal obstruction.
98.D. Absorbent antidiarrheal medications bind to diarrhea-causing bacteria to form a
nonabsorbable complex, which is then excreted in the stool.
99.A. Side effects associated with loperamide include CNS fatigue and dizziness, epigastric
pain, abdominal cramps, nausea, dry mouth, vomiting, and anorexia. Diarrhea is an
indication, not a side effect.
100. C. Diphenoxylate acts on the smooth muscle of the intestinal tract to inhibit GI
motility and excessive propulsion of the GI tract (peristalsis).
101. A. Milk of Magnesia is a saline laxative that increases osmotic pressure to draw water
into the colon.
102. C. Lactulose reduces blood ammonia levels by forcing ammonia from the blood into
the colon. It is useful in treating patients with hepatic encephalopathy.
1. Which of the following conditions can cause a hiatal hernia?

A. Increased intrathoracic pressure


B. Weakness of the esophageal muscle
C. Increased esophageal muscle pressure
D. Weakness of the diaphragmic muscle
2. Risk factors for the development of hiatal hernias are those that lead to increased abdominal
pressure. Which of the following complications can cause increased abdominal pressure?

A. Obesity
B. Volvulus
C. Constipation
D. Intestinal obstruction
3. Which of the following symptoms is common with a hiatal hernia?

A. Left arm pain


B. Lower back pain
C. Esophageal reflux
D. Abdominal cramping
4. Which of the following tests can be performed to diagnose a hiatal hernia?

A. Colonoscopy
B. Lower GI series
C. Barium swallow
D. Abdominal x-rays
5. Which of the following measures should the nurse focus on for the client with esophageal
varices?
A. Recognizing hemorrhage
B. Controlling blood pressure
C. Encouraging nutritional intake
D. Teaching the client about varices
6. Which of the following tests can be used to diagnose ulcers?

A. Abdominal x-ray
B. Barium swallow
C. Computed tomography (CT) scan
D. Esophagogastroduodenoscopy (EGD)
7. Which of the following best describes the method of action of medications, such as ranitidine
(Zantac), which are used in the treatment of peptic ulcer disease?

A. Neutralize acid
B. Reduce acid secretions
C. Stimulate gastrin release
D. Protect the mucosal barrier
8. The hospitalized client with GERD is complaining of chest discomfort that feels like heartburn
following a meal. After administering an ordered antacid, the nurse encourages the client to lie in
which of the following positions?

A. Supine with the head of the bed flat


B. On the stomach with the head flat
C. On the left side with the head of the bed elevated 30 degrees
D. On the right side with the head of the bed elevated 30 degrees.
9. The nurse is caring for a client following a Billroth II procedure. On review of the post-
operative orders, which of the following, if prescribed, would the nurse question and verify?

A. Irrigating the nasogastric tube


B. Coughing a deep breathing exercises
C. Leg exercises
D. Early ambulation
10. The nurse is providing discharge instructions to a client following gastrectomy. Which measure
will the nurse instruct the client to follow to assist in preventing dumping syndrome?

A. Eat high-carbohydrate foods


B. Limit the fluids taken with meals
C. Ambulate following a meal
D. Sit in a high-Fowlers position during meals
11. The nurse instructs the nursing assistant on how to provide oral hygiene for a client who cannot
perform this task for himself. Which of the following techniques should the nurse tell the assistant
to incorporate into the client’s daily care?

A. Assess the oral cavity each time mouth care is given and record observations
B. Use a soft toothbrush to brush the client’s teeth after each meal
C. Swab the client’s tongue, gums, and lips with a soft foam applicator every 2 hours.
D. Rinse the client’s mouth with mouthwash several times a day.
12. A client with suspected gastric cancer undergoes an endoscopy of the stomach. Which of the
following assessments made after the procedure would indicate the development of a potential
complication?

A. The client complains of a sore throat


B. The client displays signs of sedation
C. The client experiences a sudden increase in temperature
D. The client demonstrates a lack of appetite
13. A client has been diagnosed with adenocarcinoma of the stomach and is scheduled to undergo
a subtotal gastrectomy (Billroth II procedure). During pre-operative teaching, the nurse is
reinforcing information about the procedure. Which of the following explanations is most accurate?

A. The procedure will result in enlargement of the pyloric sphincter


B. The procedure will result in anastomosis of the gastric stump to the jejunum
C. The procedure will result in removal of the duodenum
D. The procedure will result in repositioning of the vagus nerve
14. After a subtotal gastrectomy, the nurse should anticipate that nasogastric tube drainage will be
what color for about 12 to 24 hours after surgery?

A. Dark brown
B. Bile green
C. Bright red
D. Cloudy white
15. After a subtotal gastrectomy, care of the client’s nasogastric tube and drainage system should
include which of the following nursing interventions?

A. Irrigate the tube with 30 ml of sterile water every hour, if needed.


B. Reposition the tube if it is not draining well
C. Monitor the client for N/V, and abdominal distention
D. Turn the machine to high suction of the drainage is sluggish on low suction.
16. Which of the following would be an expected nutritional outcome for a client who has
undergone a subtotal gastrectomy for cancer?

A. Regain weight loss within 1 month after surgery


B. Resume normal dietary intake of three meals per day
C. Control nausea and vomiting through regular use of antiemetics
D. Achieve optimal nutritional status through oral or parenteral feedings
17. The client with GERD complains of a chronic cough. The nurse understands that in a client
with GERD this symptom may be indicative of which of the following conditions?

A. Development of laryngeal cancer


B. Irritation of the esophagus
C. Esophageal scar tissue formation
D. Aspiration of gastric contents
18. Which of the following dietary measures would be useful in preventing esophageal reflux?
A. Eating small, frequent meals
B. Increasing fluid intake
C. Avoiding air swallowing with meals
D. Adding a bedtime snack to the dietary plan
19. A client is admitted to the hospital after vomiting bright red blood and is diagnosed with a
bleeding duodenal ulcer. The client develops a sudden, sharp pain in the midepigastric area along
with a rigid, boardlike abdomen. These clinical manifestations most likely indicate which of the
following?

A. An intestinal obstruction has developed


B. Additional ulcers have developed
C. The esophagus has become inflamed
D. The ulcer has perforated
20. When obtaining a nursing history on a client with a suspected gastric ulcer, which signs and
symptoms would the nurse expect to see? Select all that apply.

A. Epigastric pain at night


B. Relief of epigastric pain after eating
C. Vomiting
D. Weight loss
21. The nurse is caring for a client who has had a gastroscopy. Which of the following symptoms
may indicate that the client is developing a complication related to the procedure? Select all that
apply.

A. The client complains of a sore throat


B. The client has a temperature of 100*F
C. The client appears drowsy following the procedure
D. The client complains of epigastric pain
E. The client experiences hematemesis
22. A client with peptic ulcer disease tells the nurse that he has black stools, which he has not
reported to his physician. Based on this information, which nursing diagnosis would be appropriate
for this client?

A. Ineffective coping related to fear of diagnosis of chronic illness


B. Deficient knowledge related to unfamiliarity with significant signs and symptoms
C. Constipation related to decreased gastric motility
D. Imbalanced nutrition: Less than body requirements due to gastric bleeding
23. A client with a peptic ulcer reports epigastric pain that frequently awakens her at night, a
feeling of fullness in the abdomen, and a feeling of anxiety about her health. Based on this
information, which nursing diagnosis would be most appropriate?

A. Imbalanced Nutrition: Less than Body Requirements related to anorexia.


B. Disturbed Sleep Pattern related to epigastric pain
C. Ineffective Coping related to exacerbation of duodenal ulcer
D. Activity Intolerance related to abdominal pain
24. While caring for a client with peptic ulcer disease, the client reports that he has been nauseated
most of the day and is now feeling lightheaded and dizzy. Based upon these findings, which
nursing actions would be most appropriate for the nurse to take? Select all that apply.

A. Administering an antacid hourly until nausea subsides.


B. Monitoring the client’s vital signs
C. Notifying the physician of the client’s symptoms
D. Initiating oxygen therapy
E. Reassessing the client on an hour
25. A client is to take one daily dose of ranitidine (Zantac) at home to treat her peptic ulcer. The
nurse knows that the client understands proper drug administration of ranitidine when she says that
she will take the drug at which of the following times?

A. Before meals
B. With meals
C. At bedtime
D. When pain occurs
26. A client has been taking aluminum hydroxide 30 mL six times per day at home to treat his
peptic ulcer. He tells the nurse that he has been unable to have a bowel movement for 3 days. Based
on this information, the nurse would determine that which of the following is the most likely cause
of the client’s constipation?

A. The client has not been including enough fiber in his diet
B. The client needs to increase his daily exercise
C. The client is experiencing a side effect of the aluminum hydroxide.
D. The client has developed a gastrointestinal obstruction.
27. A client is taking an antacid for treatment of a peptic ulcer. Which of the following statements
best indicates that the client understands how to correctly take the antacid?

A. “I should take my antacid before I take my other medications.”


B. “I need to decrease my intake of fluids so that I don’t dilute the effects of my antacid.”
C. “My antacid will be most effective if I take it whenever I experience stomach pains.”
D. “It is best for me to take my antacid 1 to 3 hours after meals.”
28. The nurse is caring for a client with chronic gastritis. The nurse monitors the client, knowing
that this client is at risk for which of the following vitamin deficiencies?

A. Vitamin A
B. Vitamin B12
C. Vitamin C
D. Vitamin E
29. The nurse is reviewing the medication record of a client with acute gastritis. Which
medication, if noted on the client’s record, would the nurse question?

A. Digoxin (Lanoxin)
B. Indomethacin (Indocin)
C. Furosemide (Lasix)
D. Propranolol hydrochloride (Inderal)
30. The nurse is assessing a client 24 hours following a cholecystectomy. The nurse notes that the
T-tube has drained 750ml of green-brown drainage. Which nursing intervention is most
appropriate?

A. Notify the physician


B. Document the findings
C. Irrigate the T-tube
D. Clamp the T-tube
31. The nurse provides medication instructions to a client with peptic ulcer disease. Which
statement, if made by the client, indicates the best understanding of the medication therapy?

A. “The cimetidine (Tagamet) will cause me to produce less stomach acid.”


B. “Sucralfate (Carafate) will change the fluid in my stomach.”
C. “Antacids will coat my stomach.”
D. “Omeprazole (Prilosec) will coat the ulcer and help it heal.”
32. The client with peptic ulcer disease is scheduled for a pyloroplasty. The client asks the nurse
about the procedure. The nurse plans to respond knowing that a pyloroplasty involves:

A. Cutting the vagus nerve


B. Removing the distal portion of the stomach
C. Removal of the ulcer and a large portion of the cells that produce hydrochloric acid
D. An incision and resuturing of the pylorus to relax the muscle and enlarge the opening
from the stomach to the duodenum.
33. A client with a peptic ulcer is scheduled for a vagotomy. The client asks the nurse about the
purpose of this procedure. The nurse tells the client that the procedure:

A. Decreases food absorption in the stomach


B. Heals the gastric mucosa
C. Halts stress reactions
D. Reduces the stimulus to acid secretions
34. The nurse would assess the client experiencing an acute episode of cholecysitis for pain that is
located in the right

A. Upper quadrant and radiates to the left scapula and shoulder


B. Upper quadrant and radiates to the right scapula and shoulder
C. Lower quadrant and radiates to the umbilicus
D. Lower quadrant and radiates to the back
35. Which of the following tasks should be included in the immediate postoperative management
of a client who has undergone gastric resection?

A. Monitoring gastric pH to detect complications


B. Assessing for bowel sounds
C. Providing nutritional support
D. Monitoring for symptoms of hemorrhage
36. If a gastric acid perforates, which of the following actions should not be included in the
immediate management of the client?

A. Blood replacement
B. Antacid administration
C. Nasogastric tube suction
D. Fluid and electrolyte replacement
37. Mucosal barrier fortifiers are used in peptic ulcer disease management for which of the
following indications?

A. To inhibit mucus production


B. To neutralize acid production
C. To stimulate mucus production
D. To stimulate hydrogen ion diffusion back into the mucosa
38. When counseling a client in ways to prevent cholecystitis, which of the following guidelines is
most important?

A. Eat a low-protein diet


B. Eat a low-fat, low-cholesterol diet
C. Limit exercise to 10 minutes/day
D. Keep weight proportionate to height
39. Which of the following symptoms best describes Murphy’s sign?

A. Periumbilical eccymosis exists


B. On deep palpitation and release, pain in elicited
C. On deep inspiration, pain is elicited and breathing stops
D. Abdominal muscles are tightened in anticipation of palpation
40. Which of the following tests is most commonly used to diagnose cholecystitis?

A. Abdominal CT scan
B. Abdominal ultrasound
C. Barium swallow
D. Endoscopy
41. Which of the following factors should be the main focus of nursing management for a client
hospitalized for cholecystitis?

A. Administration of antibiotics
B. Assessment for complications
C. Preparation for lithotripsy
D. Preparation for surgery
42. A client being treated for chronic cholecystitis should be given which of the following
instructions?

A. Increase rest
B. Avoid antacids
C. Increase protein in diet
D. Use anticholinergics as prescribed
43. The client with a duodenal ulcer may exhibit which of the following findings on assessment?

A. Hematemesis
B. Malnourishment
C. Melena
D. Pain with eating
44. The pain of a duodenal ulcer can be distinguished from that of a gastric ulcer by which of the
following characteristics?

A. Early satiety
B. Pain on eating
C. Dull upper epigastric pain
D. Pain on empty stomach
45. The client has orders for a nasogastric (NG) tube insertion. During the procedure, instructions
that will assist in the insertion would be:

A. Instruct the client to tilt his head back for insertion in the nostril, then flex his neck for
the final insertion
B. After insertion into the nostril, instruct the client to extend his neck
C. Introduce the tube with the client’s head tilted back, then instruct him to keep his head
upright for final insertion
D. Instruct the client to hold his chin down, then back for insertion of the tube
46. The most important pathophysiologic factor contributing to the formation of esophageal
varices is:

A. Decreased prothrombin formation


B. Decreased albumin formation by the liver
C. Portal hypertension
D. Increased central venous pressure
47. The client being treated for esophageal varices has a Sengstaken-Blakemore tube inserted to
control the bleeding. The most important assessment is for the nurse to:

A. Check that the hemostat is on the bedside


B. Monitor IV fluids for the shift
C. Regularly assess respiratory status
D. Check that the balloon is deflated on a regular basis
48. A female client complains of gnawing epigastric pain for a few hours after meals. At times,
when the pain is severe, vomiting occurs. Specific tests are indicated to rule out:

A. Cancer of the stomach


B. Peptic ulcer disease
C. Chronic gastritis
D. Pylorospasm
49. When a client has peptic ulcer disease, the nurse would expect a priority intervention to be:
A. Assisting in inserting a Miller-Abbott tube
B. Assisting in inserting an arterial pressure line
C. Inserting a nasogastric tube
D. Inserting an I.V.
50. A 40-year-old male client has been hospitalized with peptic ulcer disease. He is being treated
with a histamine receptor antagonist (cimetidine), antacids, and diet. The nurse doing discharge
planning will teach him that the action of cimetidine is to:

A. Reduce gastric acid output


B. Protect the ulcer surface
C. Inhibit the production of hydrochloric acid (HCl)
D. Inhibit vagus nerve stimulation
Answers and Rationales
1. D. A hiatal hernia is caused by weakness of the diaphragmic muscle and increased intra-
abdominal—not intrathoracic—pressure. This weakness allows the stomach to slide into
the esophagus. The esophageal supports weaken, but esophageal muscle weakness or
increased esophageal muscle pressure isn’t a factor in hiatal hernia.
2. A. Obesity may cause increased abdominal pressure that pushes the lower portion of the
stomach into the thorax.
3. C. Esophageal reflux is a common symptom of hiatal hernia. This seems to be associated
with chronic exposure of the lower esophageal sphincter to the lower pressure of the
thorax, making it less effective.
4. C. A barium swallow with fluoroscopy shows the position of the stomach in relation to
the diaphragm. A colonoscopy and a lower GI series show disorders of the intestine.
5. A. Recognizing the rupture of esophageal varices, or hemorrhage, is the focus of nursing
care because the client could succumb to this quickly. Controlling blood pressure is also
important because it helps reduce the risk of variceal rupture. It is also important to
teach the client what varices are and what foods he should avoid such as spicy foods.
6. D. The EGD can visualize the entire upper GI tract as well as allow for tissue specimens
and electrocautery if needed. The barium swallow could locate a gastric ulcer. A CT
scan and an abdominal x-ray aren’t useful in the diagnosis of an ulcer.
7. B. Ranitidine is a histamine-2 receptor antagonist that reduces acid secretion by
inhibiting gastrin secretion.
8. C. The discomfort of reflux is aggravated by positions that compress the abdomen and
the stomach. These include lying flat on the back or on the stomach after a meal of lying
on the right side. The left side-lying position with the head of the bed elevated is most
likely to give relief to the client.
9. A. In a Billroth II procedure the proximal remnant of the stomach is anastomased to the
proximal jejunum. Patency of the NG tube is critical for preventing the retention of
gastric secretions. The nurse should never irrigate or reposition the gastric tube after
gastric surgery, unless specifically ordered by the physician. In this situation, the nurse
would clarify the order.
10.B. The nurse should instruct the client to decrease the amount of fluid taken at meals and
to avoid high carbohydrate foods including fluids such as fruit nectars; to assume a low-
Fowler’s position during meals; to lie down for 30 minutes after eating to delay gastric
emptying; and to take antispasmidocs as prescribed.
11.B. A soft toothbrush should be used to brush the client’s teeth after each meal and more
often as needed. Mechanical cleaning is necessary to maintain oral health, simulate
gingiva, and remove plaque. Assessing the oral cavity and recording observations is the
responsibility of the nurse, not the nursing assistant. Swabbing with a safe foam
applicator does not provide enough friction to clean the mouth. Mouthwash can be a
drying irritant and is not recommended for frequent use.
12.C. The most likely complication of an endoscopic procedure is perforation. A sudden
temperature spike with 1 to 2 hours after the procedure is indicative of a perforation and
should be reported immediately to the physician. A sore throat is to be anticipated after
an endoscopy. Clients are given sedatives during the procedure, so it is expected that
they will display signs of sedation after the procedure is completed. A lack of appetite
could be the result of many factors, including the disease process.
13.B. A Billroth II procedure bypasses the duodenum and connects the gastric stump
directly to the jejunum. The pyloric sphincter is removed, along with some of the
stomach fundus.
14.A. About 12 to 24 hours after a subtotal gastrectomy, gastric drainage is normally
brown, which indicates digested blood. Bile green or cloudy white drainage is not
expected during the first 12 to 24 hours after a subtotal gastrectomy. Drainage during the
first 6 to 12 hours contains some bright red blood, but large amounts of blood or
excessively bloody drainage should be reported to the physician promptly.
15.C. Nausea, vomiting, or abdominal distention indicated that gas and secretions are
accumulating within the gastric pouch due to impaired peristalsis or edema at the
operative site and may indicate that the drainage system is not working properly. Saline
solution is used to irrigate nasogastric tubes. Hypotonic solutions such as water increase
electrolyte loss. In addition, a physician’s order is needed to irrigate the NG tube,
because this procedure could disrupt the suture line. After gastric surgery, only the
surgeon repositions the NG tube because of the danger of rupturing or dislodging the
suture line. The amount of suction varies with the type of tube used and is ordered by the
physician. High suction may create too much tension on the gastric suture line.
16.D. An appropriate expected outcome is for the client to achieve optimal nutritional status
through the use of oral feedings or total parenteral nutrition (TPN). TPN may be used to
supplement oral intake, or it may be used alone if the client cannot tolerate oral feedings.
The client would not be expected to regain lost weight within 1 month after surgery or to
tolerate a normal dietary intake of three meals per day. Nausea and vomiting would not
be considered an expected outcome of gastric surgery, and regular use of antiemetics
would not be anticipated.
17.D. Clients with GERD can develop pulmonary symptoms such as coughing, wheezing,
and dyspnea that are caused by the aspiration of gastric contents. GERD does not
predispose the client to the development of laryngeal cancer. Irritation of the esophagus
and esophageal scar tissue formation can develop as a result of GERD. However, GERD
is more likely to cause painful and difficult swallowing.
18.A. Esophageal reflux worsens when the stomach is overdistended with food. Therefore,
an important measure is to eat small, frequent meals. Fluid intake should be decreased
during meals to reduce abdominal distention. Avoiding air swallowing does not prevent
esophageal reflux. Food intake in the evening should be strictly limited to reduce the
incidence of nighttime reflux, so bedtime snacks are not recommended.
19.D. The body reacts to perforation of an ulcer by immobilizing the area as much as
possible. This results in boardlike muscle rigidity, usually with extreme pain. Perforation
is a medical emergency requiring immediate surgical intervention because peritonitis
develops quickly after perforation. An intestinal obstruction would not cause
midepigastric pain. Esophageal inflammation or the development of additional ulcers
would not cause a rigid, boardlike abdomen.
20.C and D. Vomiting and weight loss are common with gastric ulcers. Clients with a
gastric ulcer are most likely to complain of a burning epigastric pain that occurs about
one hour after eating. Eating frequently aggravates the pain. Clients with duodenal
ulcers are more likely to complain about pain that occurs during the night and is
frequently relieved by eating.
21.B, D, and E. Following a gastroscopy, the nurse should monitor the client for
complications, which include perforation and the potential for aspiration. An elevated
temperature, complaints of epigastric pain, or the vomiting of blood (hematemesis) are
all indications of a possible perforation and should be reported promptly. A sore throat is
a common occurrence following a gastroscopy. Clients are usually sedated to decrease
anxiety and the nurse would anticipate that the client will be drowsy following the
procedure.
22.B. Black, tarry stools are an important warning sign of bleeding in peptic ulcer disease.
Digested blood in the stomach causes it to be black. The odor of the stool is very stinky.
Clients with peptic ulcer disease should be instructed to report the incidence of black
stools promptly to their physician.
23.B. Based on the data provided, the most appropriate nursing diagnosis would be
Disturbed Sleep pattern. A client with a duodenal ulcer commonly awakens at night with
pain. The client’s feelings of anxiety do not necessarily indicate that she is coping
ineffectively.
24.B and C. The symptoms of nausea and dizziness in a client with peptic ulcer disease may
be indicative of hemorrhage and should not be ignored. The appropriate nursing actions
at this time are for the nurse to monitor the client’s vital signs and notify the physician of
the client’s symptoms. To administer an antacid hourly or to wait one hour to reassess
the client would be inappropriate; prompt intervention is essential in a client who is
potentially experiencing a gastrointestinal hemorrhage. The nurse would notify the
physician of assessment findings and then initiate oxygen therapy if ordered by the
physician.
25.C. Ranitidine blocks secretion of hydrochloric acid. Clients who take only one daily
dose of ranitidine are usually advised to take it at bedtime to inhibit nocturnal secretion
of acid. Clients who take the drug twice a day are advised to take it in the morning and
at bedtime.
26.C. It is most likely that the client is experiencing a side effect of the antacid. Antacids
with aluminum salt products, such as aluminum hydroxide, form insoluble salts in the
body. These precipitate and accumulate in the intestines, causing constipation.
Increasing dietary fiber intake or daily exercise may be a beneficial lifestyle change for
the client but is not likely to relieve the constipation caused by the aluminum hydroxide.
Constipation, in isolation from other symptoms, is not a sign of bowel obstruction.
27.D. Antacids are most effective if taken 1 to 3 hours after meals and at bedtime. When an
antacid is taken on an empty stomach, the duration of the drug’s action is greatly
decreased. Taking antacids 1 to 3 hours after a meal lengthens the duration of action,
thus increasing the therapeutic action of the drug. Antacids should be administered about
2 hours after other medications to decrease the chance of drug interactions. It is not
necessary to decrease fluid intake when taking antacids.
28.B. Chronic gastritis causes deterioration and atrophy of the lining of the stomach,
leading to the loss of the functioning parietal cells. The source of the intrinsic factor is
lost, which results in the inability to absorb vitamin B12. This leads to the development of
pernicious anemia.
29.B. Indomethacin (Indocin) is a NSAID and can cause ulceration of the esophagus,
stomach, duodenum, or small intestine. Indomethacin is contraindicated in a client with
GI disorders.
30.B. Following cholecystectomy, drainage from the T-tube is initially bloody and then
turns to green-brown. The drainage is measured as output. The amount of expected
drainage will range from 500 to 1000 ml per day. The nurse would document the output.
31.A. Cimetidine (Tagamet), a histamine H2 receptor antagonist, will decrease the secretion
of gastric acid. Sucralfate (Carafate) promotes healing by coating the ulcer. Antacids
neutralize acid in the stomach. Omeprazole (Prilosec) inhibits gastric acid secretion.
32.D. An incision and resuturing of the pylorus to relax the muscle and enlarge the opening
from the stomach to the duodenum describes the procedure for a pyloroplasty. A
vagotomy involves cutting the vagus nerve. A subtotal gastrectomy involves removing
the distal portion of the stomach. A Billroth II procedure involves removal of the ulcer
and a large portion of the tissue that produces hydrochloric acid.
33.D. A vagotomy, or cutting the vagus nerve, is done to eliminate parasympathetic
stimulation of gastric secretion.
34.B. During an acute “gallbladder attack,” the client may complain of severe right upper
quadrant pain that radiates to the right scapula and shoulder. This is governed by the
pattern on dermatones in the body.
35.D. The client should be monitored closely for signs and symptoms of hemorrhage, such
as bright red blood in the nasogastric tube suction, tachycardia, or a drop in blood
pressure. Gastric pH may be monitored to evaluate the need for histamine-2 receptor
antagonists. Bowel sounds may not return for up to 72 hours postoperatively. Nutritional
needs should be addressed soon after surgery.
36.B. Antacids aren’t helpful in perforation. The client should be treated with antibiotics as
well as fluid, electrolyte, and blood replacement. NG tube suction should also be
performed to prevent further spillage of stomach contents into the peritoneal cavity.
37.C. The mucosal barrier fortifiers stimulate mucus production and prevent hydrogen ion
diffusion back into the mucosa, resulting in accelerated ulcer healing. Antacids
neutralize acid production.
38.D. Obesity is a known cause of gallstones, and maintaining a recommended weight will
help protect against gallstones. Excessive dietary intake of cholesterol is associated with
the development of gallstones in many people. Dietary protein isn’t implicated in
cholecystitis. Liquid protein and low-calorie diets (with rapid weight loss of more than 5
lb [2.3kg] per week) are implicated as the cause of some cases of cholecystitis. Regular
exercise (30 minutes/three times a week) may help reduce weight and improve fat
metabolism. Reducing stress may reduce bile production, which may also indirectly
decrease the chances of developing cholecystitis.
39.C. Murphy’s sign is elicited when the client reacts to pain and stops breathing. It’s a
common finding in clients with cholecystitis. Periumbilical ecchymosis, Cullen’s sign, is
present in peritonitis. Pain on deep palpation and release is rebound tenderness.
Tightening up abdominal muscles in anticipation of palpation is guarding.
40.B. An abdominal ultrasound can show if the gallbladder is enlarged, if gallstones are
present, if the gallbladder wall is thickened, or if distention of the gallbladder lumen is
present. An abdominal CT scan can be used to diagnose cholecystitis, but it usually isn’t
necessary. A barium swallow looks at the stomach and the duodenum. Endoscopy looks
at the esophagus, stomach, and duodenum.
41.B. The client with acute cholecystitis should first be monitored for perforation, fever,
abscess, fistula, and sepsis. After assessment, antibiotics will be administered to reduce
the infection. Lithotripsy is used only for a small percentage of clients. Surgery is
usually done after the acute infection has subsided.
42.D. Conservative therapy for chronic cholecystitis includes weight reduction by
increasing physical activity, a low-fat diet, antacid use to treat dyspepsia, and
anticholinergic use to relax smooth muscles and reduce ductal tone and spasm, thereby
reducing pain.
43.C. The client with a duodenal ulcer may have bleeding at the ulcer site, which shows up
as melena (black tarry poop). The other findings are consistent with a gastric ulcer.
44.D. Pain on empty stomach is relieved by taking foods or antacids. The other symptoms
are those of a gastric ulcer.
45.A. NG insertion technique is to have the client first tilt his head back for insertion into
the nostril, then to flex his neck forward and swallow. Extension of the neck will impede
NG tube insertion.
46.C. As the liver cells become fatty and degenerate, they are no longer able to
accommodate the large amount of blood necessary for homeostasis. The pressure in the
liver increases and causes increased pressure in the venous system. As the portal
pressure increases, fluid exudes into the abdominal cavity. This is called ascites.
47.C. The respiratory system can become occluded if the balloon slips and moves up the
esophagus, putting pressure on the trachea. This would result in respiratory distress and
should be assessed frequently. Scissors should be kept at the bedside to cut the tube if
distress occurs. This is a safety intervention.
48.B. Peptic ulcer disease is characteristically gnawing epigastric pain that may radiate to
the back. Vomiting usually reflects pyloric spasm from muscular spasm or obstruction.
Cancer would not evidence pain or vomiting unless the pylorus was obstructed.
49.C. An NG tube insertion is the most appropriate intervention because it will determine
the presence of active GI bleeding. A Miller-Abbott tube is a weighted, mercury-filled
ballooned tube used to resolve bowel obstructions. There is no evidence of shock or
fluid overload in the client; therefore, an arterial line is not appropriate at this time and
an IV is optional.
50.A. These drugs inhibit action of histamine on the H2 receptors of parietal cells, thus
reducing gastric acid output.

You might also like